Vous êtes sur la page 1sur 63

CARDIO PACKRAT

Study online at quizlet.com/_23pihf

1. History & Physical/Cardiology Explanations


Which of the following conditions would cause a positive (u) A. Left ventricular failure results in the back-up of blood into
Kussmaul's sign on physical examination? the left atrium and then the pulmonary system so it would not
Answers be associated with Kussmaul's sign.
A. Left ventricular failure (u) B. Pulmonary edema primarily results in increased
B. Pulmonary edema pulmonary pressures rather than having effects on the venous
C. Coarctation of the aorta inflow into the heart.
D. Constrictive pericarditis (u) C. Coarctation of the aorta primarily affects outflow from the
heart due to the stenosis resulting in delayed and decreased
femoral pulses; it has no effect on causing Kussmaul's sign.
(c) D. Kussmaul's sign is an increase rather than the normal
decrease in the CVP during inspiration. It is most often caused
by severe right-sided heart failure; it is a frequent finding in
patients with constrictive pericarditis or right ventricular
infarction.
2. History & Physical/Cardiology Explanations
Anginal chest pain is most commonly described as which of the (u) A. Pain changing with position or respiration is suggestive of
following? pericarditis.
Answers (c) B. Myocardial ischemia is often experienced as a sensation of
A. Pain changing with position or respiration discomfort lasting 5-15 minutes, described as dull, aching or
B. A sensation of discomfort pressure.
C. Tearing pain radiating to the back (u) C. Tearing pain with radiation to the back represents aortic
D. Pain lasting for several hours dissection.
(u) D. Chest pain lasting for several hours is more suggestive for
myocardial infarction.
3. History & Physical/Cardiology Explanations
Eliciting a history from a patient presenting with dyspnea due to (c) A. The amount of activity that precipitates dyspnea should
early heart failure the severity of the dyspnea should be be quantified in the history.
quantified by (u) B. Orthopnea or paroxysmal nocturnal dyspnea can be
Answers quantified by how many pillows a patient needs to sleep on to
A. amount of activity that precipitates it. be comfortable.
B. how many pillows they sleep on at night. (u) C. How long dyspnea takes to resolve or associated
C. how long it takes the dyspnea to resolve. comorbidities has no bearing on quantifying the severity of
D. any associated comorbidities. dyspnea.
(u) D. See answer C above.
4. History & Physical/Cardiology Explanations
A 25 year-old female presents with a three-day history of chest (u) A. Pulsus paradoxus is a classic finding for cardiac
pain aggravated by coughing and relieved by sitting. She is febrile tamponade.
and a CBC with differential reveals leukocytosis. Which of the (u) B. Localized crackles are associated with pneumonia and
following physical exam signs is characteristic of her problem? consolidation, not pericarditis.
Answers (c) C. Pericardial friction rub is characteristic of an inflammatory
A. Pulsus paradoxus pericarditis.
B. Localized crackles (u) D. Wheezing is characteristic for pulmonary disorders, such
C. Pericardial friction rub as asthma.
D. Wheezing
5. History & Physical/Cardiology Explanations
A 65 year-old white female presents with dilated tortuous veins on (u) A. Patients with deep venous thrombosis (DVT) may
the medial aspect of her lower extremities. Which of the following present with complaints of pain in the calf with ambulation.
would be the most common initial complaint? Secondary varicosities may result from DVT's.
Answers (c) B. Dull aching heaviness or a feeling of fatigue brought on
A. Pain in the calf with ambulation by periods of standing is the most common complaint of
B. Dull aching heaviness brought on by periods of standing patients presenting initially with varicosities.
C. Brownish pigmentation above the ankle (u) C. Stasis Dermatitis and edema are most suggestive of
D. Edema in the lower extremities chronic venous insufficiency.
(u) D. See C for explanation.
6. History & Physical/Cardiology Explanations
A 22 year-old male received a stab wound in the chest an hour ago. (u) A. Pulmonary edema may result with low output states as
The diagnosis of pericardial tamponade is strongly supported by seen with myocardial contusions, but it is not strongly
the presence of suggestive of tamponade.
Answers (u) B. Wide pulse pressure is seen in conditions of high stroke
A. pulmonary edema. volume such as aortic insufficiency or hyperthyroidism.
B. wide pulse pressure. Narrow pulse pressure is seen with cardiac tamponade.
C. distended neck veins. (c) C. Cardiac compression will manifest with distended neck
D. an early diastolic murmur. veins and cold clammy skin.
(u) D. The onset of diastolic murmur is suggestive of valvular
disease, not tamponade.
7. Diagnostic Studies/Cardiology Explanations
Cardiac nuclear scanning is done to detect (u) A. An EKG is used to determine electrical conduction
Answers abnormalities.
A. electrical conduction abnormalities. (u) B. An echocardiogram is a non-invasive test used to
B. valvular abnormalities. determine valvular abnormalities and wall motion.
C. ventricular wall dysfunction. (c) C. Visualization of the cardiac wall can be done with cardiac
D. coronary artery patency/occlusion. nuclear scanning. This is done to determine hypokinetic areas
from akinetic areas.
(u) D. Patency or occlusion is assessed with cardiac
catheterization (invasive).
8. Diagnostic Studies/Cardiology (u) A. Electrical conduction will not assess for mural thrombi.
A 72 year-old male with a new diagnosis of congestive heart (u) B. A chest x-ray will not visualize the left atria and ventricles to
failure and atrial fibrillation, develops episodes of assess for mural thrombi.
hemodynamic compromise secondary to increased (c) C. Transesophageal echocardiography allows for determination of
ventricular rate. A decision to perform elective mural thrombi that may have resulted from atrial fibrillation.
cardioversion is made and the patient is anticoagulated (u) D. C-reactive protein is not going to give you any information
with heparin. Which test should be ordered to assess for regarding thrombi. This test is used to identify the presence of
atrial or ventricular mural thrombi? inflammation.

A. Electrocardiogram
B. Chest x-ray
C. Transesophageal Echocardiogram
D. C-reactive protein
9. Diagnostic Studies/Cardiology Explanations
A 64 year-old patient with known history of type 1 diabetes (u) A. See B for explanation.
mellitus for 50 years has developed pain radiating from the (c) B. Given the patient's long history of type 1 diabetes mellitus the
right buttock to the calf. Patient states that the pain is patient most likely has vascular occlusive disease. Evaluation of arterial
made worse with walking and climbing stairs. Based upon blood flow is assessed using the duplex scanner. X-ray of the L/S spine
this history which of the following would be the most and right hip while not harmful may give information regarding bony
appropriate test to order? structures. Venous Doppler ultrasound will not give information of
Answers arterial perfusion.
A. Venogram (u) C. See B for explanation.
B. Arterial duplex scanning (u) D. See B for explanation.
C. X-ray of the right hip and L/S spine
D. Venous Doppler ultrasound
10. Diagnostic Studies/Cardiology Explanations
A 36 year-old male complains of occasional episodes of (c) A. Holter monitoring is a non-invasive test done to obtain a
"heart fluttering". The patient describes these episodes as continuous monitoring of the electrical activity of the heart. This can
frequent, short-lived and episodic. He denies any help to detect cardiac rhythm disturbances that can correlate with the
associated chest pain. Based on this information, which patient symptoms. Cardiac catheterization is an invasive procedure
one of the following tests would be the most appropriate done to assess coronary artery disease. Stress testing and cardiac
to order? nuclear scanning are non-invasive testing maneuvers done to assess
Answers coronary artery disease.
A. Holter monitor (u) B. See A for explanation.
B. Cardiac catheterization (u) C. See A for explanation.
C. Stress testing (u) D. See A for explanation.
D. Cardiac nuclear scanning
11. Diagnostic Studies/Cardiology Explanations
A patient with a mitral valve replacement was placed post- (u) A. PTT is a reflection of the intrinsic clotting system and is used to
operatively on warfarin (Coumadin) for anticoagulation monitor heparin administration.
prophylaxis. To monitor this drug for its effectiveness, (c) B. PT-INR is a reflection of the extrinsic and common pathway
what test would be used? clotting system. Coumadin interferes with Vitamin K synthesis which is
Answers needed in the manufacture of factors II, VII, IX, X which are part of the
A. PTT extrinsic clotting pathway.
B. PT-INR (u) C. Platelet aggregation tests are utilized to assess platelet
C. Platelet aggregation dysfunction.
D. Bleeding time (u) D. Bleeding time is used to assess platelet function.
12. Diagnosis/Cardiology Explanations
A 64 year-old male, with a long history of COPD, presents with increasing fatigue over the last (c) A. Signs of right
three months. The patient has stopped playing golf and also complains of decreased appetite, ventricular failure are fluid
chronic cough and a bloated feeling. Physical examination reveals distant heart sounds, retention i.e. edema,
questionable gallop, lungs with decreased breath sounds at lung bases and the abdomen reveals hepatic congestion and
RUQ tenderness with the liver two finger-breadths below the costal margin, the extremities show possibly ascites.
2+/4+ pitting edema. Labs reveal the serum creatinine level 1.6 mg/dl, BUN 42 mg/dl, liver function (u) B. See A for explanation.
test's mildly elevated and the CBC to be normal. Which of the following is the most likely (u) C. See A for explanation.
diagnosis? (u) D. See A for explanation.
Answers
A. Right ventricular failure
B. Pericarditis
C. Exacerbation of COPD
D. Cirrhosis
13. Diagnosis/Cardiology Explanations
A 56 year-old male with a known history of polycythemia suddenly complains of pain and (u) A. See B for explanation.
paresthesia in the left leg. Physical examination reveals the left leg is cool to the touch and the (c) B. Arterial thrombosis
toes are cyanotic. The popliteal pulse is absent by palpation and Doppler. The femoral pulse is has occurred and is
absent by palpation but weak with Doppler. The right leg and upper extremities has 2+/4+ pulses evidenced by the loss of the
throughout. Given these findings what is the most likely diagnosis? popliteal and dorsalis pedis
Answers pulse. This is a surgical
A. Venous thrombosis emergency. Venous
B. Arterial thrombosis occlusion and
C. Thromboangiitis obliterans thrombophlebitis do not
D. Thrombophlebitis result in loss of arterial
pulse.
(u) C. See B for explanation.
(u) D. See B for explanation.
14. Diagnosis/Cardiology Explanations
A 48 year-old male with a known history of hypertension is brought to the ED complaining of (u) A. See D for explanation.
headache, general malaise, nausea and vomiting. The patient currently takes nifedipine (u) B. See D for explanation.
(Procardia)90mg XL every day and atenolol (Tenormin) 50 mg every day. Vital signs reveal (u) C. Pseudotumor cerebri
temperature 98.6F, pulse 72/minute, respiratory rate 20/minute, and the blood pressure is presents with papilledema,
168/120 mmHg. BP reading taken every 15 minutes from the time of admission reveal the systolic but not hypertension and is
to run from 176 to 186 mmHg and the diastolic to run from 135 to 150 mmHg. Physical more common in young
examination reveals papilledema bilaterally. There are no renal bruits noted. The EKG is normal. females.
Based upon this presentation, what is the most likely diagnosis? (c) D. Malignant HTN is
Answers characterized by diastolic
A. Meningitis reading greater than 140
B. Secondary hypertension mm Hg with evidence of
C. Pseudotumor cerebri target organ damage.
D. Malignant hypertension
15. Diagnosis/Cardiology Explanations
A 55 year-old male is seen in follow-up for a complaint of chest pain. Patient states (u) A. Pain does not resolve in an acute
that he has had this chest pain for about one year now. The patient further states that MI, it gradually gets worse.
the pain is retrosternal with radiation to the jaw. "It feels as though a tightness, or (u) B. Pain typically occurs at rest is one of
heaviness is on and around my chest". This pain seems to come on with exertion the hallmarks of Prinzmetal variant
however, over the past two weeks he has noticed that he has episodes while at rest. If angina. This patient has just started to
the patient remains non- active the pain usually resolves in 15-20 minutes. Patient has develop pain at rest.
a 60-pack year smoking history and drinks a martini daily at lunch. Patient appears (u) C. Pain in stable angina is relieved
overweight on inspection. Based upon this history what is the most likely diagnosis? with rest and usually resolves within 10
Answers minutes. angina does not have pain at
A. Acute myocardial infarction rest.
B. Prinzmetal variant angina (c) D. Pain in unstable angina is
C. Stable angina precipitated by less effort than before or
D. Unstable angina occurs at rest.
16. Diagnosis/Cardiology Explanations
Stable (c) A. The scenario presented here is
A 60 year-old male is brought to the ED complaining of severe onset of chest pain and typical of an ascending aortic dissection.
intrascapular pain. The patient states that the pain feels as though "something is In an acute myocardial infarction the
ripping and tearing". The patient appears shocky; the skin is cool and clammy. The pain builds up gradually. Cardiac
patient has an impaired sensorium. Physical examination reveals a loud diastolic tamponade may occur with a dissection
murmur and variation in blood pressure between the right and left arm. Based upon into the pericardial space; syncope is
this presentation what is the most likely diagnosis? usually seen with this occurrence.
Answers Pulmonary embolism is usually associated
A. Aortic dissection with dyspnea along with chest pain.
B. Acute myocardial infarction (u) B. See A for explanation.
C. Cardiac tamponade (u) C. See A for explanation.
D. Pulmonary embolism (u) D. See A for explanation.
17. Diagnosis/Cardiology Explanations
A 42 year-old male is brought into the ED with a complaint of chest pain. The pain (u) A. See C for explanation.
comes on suddenly without exertion and lasts anywhere from 10-20 minutes. The (u) B. See C for explanation.
patient has experienced this on three previous occasions. Today the patient (c) C. Wolff-Parkinson-White syndrome
complains of light- headedness with the chest pain lasting longer. Vital signs T-99.3F hallmarks on EKG include a shorten PR
oral, P-106/minute and regular, R-22/minute, BP 146/86 mm Hg. EKG reveals sinus interval, widened QRS, and delta waves.
rhythm with a rate of 100. Intervals are PR = 0.06 seconds, QRS = 0.12 seconds. A delta Sinus tachycardia has a normal PR
wave is noted in many leads. Based upon this information what is the most likely interval and no delta waves. PSVT usually
diagnosis? has a retrograde P wave or it may be
Answers buried in the QRS complex.
A. Sinus tachycardia (u) D. Ventricular tachycardia has a
B. Paroxysmal supraventricular tachycardia widened QRS as it originates in the
C. Wolff-Parkinson-White syndrome ventricles.
D. Ventricular tachycardia
18. Diagnosis/Cardiology Explanations
A 63 year-old male is admitted to the hospital with an exacerbation of (u) A. Atrial fibrillation is an irregularly, irregular
COPD. The electrocardiogram shows an irregularly, irregular rhythm at a rhythm with no definable P waves.
rate of 120/minute with at least three varying P wave morphologies. These (c) B. Multifocal atrial tachycardia is seen most
electrocardiogram findings are most suggestive of commonly in patients with COPD. Electrocardiogram
Answers findings include an irregularly, irregular rhythm
A. atrial fibrillation. with a varying PR interval and various P wave
B. multifocal atrial tachycardia. morphologies (Three or more foci).
C. atrioventricular junctional rhythm. (u) C. Atrioventricular junctional rhythm is an escape
D. third degree heart block. rhythm, because of depressed sinus node function,
with a ventricular rate between 40-60/minute.
(u) D. Third degree heart block presents with a wide
QRS at a rate less than 50/minute and blocked atrial
impulses.
19. Health Maintenance/Cardiology Explanations
A 72 year-old female is being discharged from the hospital following an (u) A. Warfarin is not indicated since there is no role
acute anterolateral wall myocardial infarction. While in the hospital the for anticoagulation in this patient.
patient has not had any dysrhythmias or hemodynamic compromise. (c) B. ACE inhibitors have been shown to decrease
Which of the following medications should be a part of her d/c meds? left ventricular hypertrophy and remodeling to allow
for a greater ejection fraction.
A. Warfarin (Coumadin) (u) C. The patient does not have any dysrhythmias
B. Captopril (Capoten) so Lanoxin is not indicated.
C. Digoxin (Lanoxin) (u) D. The patient does not have any hemodynamic
D. Furosemide (Lasix) compromise or indicators of CHF.
20. Health Maintenance/Cardiology Explanations
A 44 year-old male with a known history of rheumatic fever at age 7 and (h) A. See C for explanation.
heart murmur is scheduled to undergo a routine dental cleaning. The (u) B. See C for explanation.
murmur is identified as an opening snap murmur. Patient has no known (c) C. These are the current recommendations from
drug allergies. What should this patient receive for antibiotic prophylaxis the American Heart Association if the patient is not
prior to the dental cleaning? allergic to penicillin.
Answers (u) D. See C for explanation.
A. This patient does not require antibiotic prophylaxis for a routine dental
cleaning.
B. This should receive Pen VK 250 mg p.o. QID for 10 days after the
procedure.
C. This patient should receive Amoxicillin 3.0 gms. p.o. 1 hour before the
procedure and then 1.5 gm. 6 hours after the procedure.
D. This patient should receive Erythromycin 250 mg QID for 1 day before
the procedure and then 10 days after the procedure.
21. Health Maintenance/Cardiology Explanations
A 36 year-old female presents for a refill of her oral contraceptives. She (c) A. Women over age 35 who smoke are at
admits to smoking one pack of cigarettes per day. She should be counseled increased risk for the development of venous
with regard to her risk of thrombosis.
Answers (u) B. Varicose veins are the result of pressure
A. venous thrombosis. overload on incompetent veins and not due to the
B. varicose veins. use of oral contraceptives.
C. atherosclerosis. (a) C. The defined risks of atherosclerosis includes
D. peripheral edema. smoking, but does not include the use of oral
contraceptives.
(u) D. There is no relationship between the use of
oral contraceptives and the development of
peripheral edema.
22. Health Maintenance/Cardiology Explanations
A 68 year-old female comes to the office for an annual physical examination. Her past (u) A. See C for explanation.
medical history is significant for a 40-pack year cigarette smoking history. She takes no (u) B. See C for explanation.
medications and has not been hospitalized for any surgery. Family medical history (c) C. This patient has 2 identifiable risk
reveals that her mother is living, age 87, in good health without medical problems. Her factors based upon the information
father is deceased at age 45 from a motor vehicle crash. She has two siblings that are provided. These include her age 68 and
alive and well. From this information, how many identifiable risk factors for her history of cigarette smoking.
cardiovascular heart disease exist in this patient? (u) D. See C for explanation.
Answers
A. 0
B. 1
C. 2
D. 3
23. Clinical Intervention/Cardiology Explanations
Following an acute anterolateral myocardial wall infarction two days ago, a patient (c) A. Free wall rupture is a complication
suddenly develops hemodynamic deterioration without EKG changes occurring. What that occurs within 72 hours of infarction.
complication can explain this scenario? It is seen mainly in Q wave transmural
Answers and lateral wall infarctions.
A. Free wall rupture (u) B. See A for explanation.
B. CVA (u) C. See A for explanation. Atrial
C. Atrial fibrillation fibrillation would have EKG evidence of
D. Sick sinus syndrome irregularly, irregular rate and rhythm.
(u) D. Sick sinus syndrome would have
EKG evidence of decreased rate and
loss of P waves.
24. Clinical Intervention/Cardiology Explanations
A 48 year-old male with a history of coronary artery disease and two myocardial (h) A. See C for explanation.
infarctions complains of shortness of breath at rest and 2-pillow orthopnea. His oxygen (h) B. See C for explanation.
saturation is 85% on room air. The patient denies any prior history of symptoms. The (c) C. An elevated BNP is seen in a
patient denies smoking. Results of a beta-natriuretic peptide (BNP) are elevated. What situation where there is increased
should be your next course of action for this patient? pressure in the ventricle during
Answers diastole. This is representative of the left
A. Send him home on 20 mg furosemide (Lasix) p.o. every day and recheck in one week ventricle being stretched excessively
B. Send him home on clarithromycin (Biaxin) 500 mg p.o. BID and recheck in 1 week when a patient has CHF. Sending a
C. Admit to the hospital for work up of left ventricular dysfunction patient home would be inappropriate
D. Admit to the hospital for work up of pneumonia in this case.
(u) D. See C for explanation.
25. Clinical Intervention/Cardiology Explanations
A 48 year-old male presents to the ED with complaints of chest pressure, (c) A. The standard of care for the management of acute
dyspnea on exertion, and diaphoresis that has been present for the last ST-segment elevation MI is coronary artery
one hour. Electrocardiogram reveals normal sinus rhythm at 92/minute revascularization. This patient is diagnosed with an ST-
along with ST segment elevation in leads V3-V5. Initial cardiac enzymes segment elevation MI based upon his history and EKG
are normal. What is the next most appropriate step in the management findings. Cardiac enzymes are normal because of the
of this patient? early presentation of this patient.
Answers (u) B. Although this patient will be admitted to the
A. Coronary artery revascularization hospital, this patient needs to have acute management
B. Admission for medical management of the myocardial infarction without delay.
C. Administer lidocaine (h) C. Prophylactic lidocaine has been shown to increase
D. Administer nitrates morbidity and mortality from acute MI when used in
this setting.
(u) D. Although pain control is a goal for patients with
acute MI, it is not the essential medication that will
impact this patient's care to the greatest degree.
26. Clinical Intervention/Cardiology Explanations
An unresponsive patient is brought to the ED by ambulance. He is in (u) A. Adenosine is used to treat PSVT.
ventricular tachycardia with a heart rate of 210 beats/min and a blood (c) B. The first step in treatment of unstable ventricular
pressure of 70/40 mmHg. The first step in treatment is to tachycardia with a pulse is to cardiovert using a 100 J
Answers countershock.
A. administer IV adenosine. (u) C. See B for explanation.
B. DC cardiovert. (u) D. Overdrive pacing is indicated in Torsades de
C. administer IV lidocaine. Pointes.
D. apply overdrive pacer.
27. Clinical Therapeutics/Cardiology Explanations
Which of the following antiarrhythmic drugs can be associated with (u) A. See B for explanation.
hyper- or hypothyroidism following long-term use? (c) B. Amiodarone is structurally related to thyroxine
Answers and contains iodine, which can induce a hyper- or
A. Quinidine hypothyroid state.
B. Amiodarone (u) C. See B for explanation.
C. Digoxin (u) D. See B for explanation.
D. Verapamil
28. Clinical Therapeutics/Cardiology Explanations
Which of the following hypertensive emergency drugs has the potential (c) A. Sodium nitroprusside metabolization results in
for developing cyanide toxicity? cyanide ion production. It can be treated with sodium
Answers thiosulfite, which combines with the cyanide ion to form
A. Sodium nitroprusside (Nipride) thiocyanate, which is nontoxic.
B. Diazoxide (Hyperstat) (u) B. See A for explanation.
C. Labetalol (Normodyne) (u) C. See A for explanation.
D. Alpha-methyldopa (Aldomet) (u) D. See A for explanation.
29. Clinical Therapeutics/Cardiology Explanations
Contraindications to beta blockade following an acute myocardial (c) A. Beta blockade is contraindicated in second and
infarction include which of the following? third heart block.
Answers (u) B. Beta blockade has been proven to be beneficial in
A. Third degree A-V block sinus tachycardia, hypertension and in atrial fib/flutter
B. Sinus tachycardia with a rapid ventricular response.
C. Hypertension (u) C. See B for explanation.
D. Rapid ventricular response to Atrial fibrillation/flutter (u) D. See B for explanation.
30. Clinical Therapeutics/Cardiology Explanations
A 74 year-old male is diagnosed with pneumonia. The physician (u) A. See C for explanation.
assistant should ensure the patient is not on which of the following (u) B. See C for explanation.
before starting therapy with clarithromycin (Biaxin)? (c) C. Statins are known to interact with the macrolides as
Answers they may cause prolonged QT interval, myopathy and
A. Lisinopril (Zestril) rhabdomyolysis.
B. Furosemide (Lasix) (u) D. See C for explanation.
C. Simvastatin (Zocor)
D. Dipyridamole (Persantine)
31. Clinical Therapeutics/Cardiology Explanations
According to the recent JNC VII guidelines, a 34 year-old male who has (u) A. Beta blockers could potentially be harmful in a
type 1 diabetes mellitus and hypertension should be started on patient with diabetes mellitus. Use a cardioselective beta-
which type of antihypertensive agent? blocker to reduce the incidence of hypoglycemia.
Answers (u) B. See C for explanation.
A. Beta-blocker (c) C. ACE inhibitors are effective in young patients. They
B. Loop diuretic are capable of providing protection to the kidney
C. ACE inhibitor especially in diabetes mellitus.
D. Thiazide diuretic (u) D. See C for explanation.
32. Clinical Therapeutics/Cardiology Explanations
Which of the following beta-adrenergic blocking agents has (u) A. Propranolol and timolol are nonselective beta-
cardioselectivity for primarily blocking beta-1 receptors? adrenergic antagonists.
Answers (u) B. See A for explanation.
A. Propranolol (Inderal) (c) C. Metoprolol is selective for beta-1 antagonists
B. Timolol (Blocadren) (u) D. Pindolol is an antagonist with partial agonist activity.
C. Metoprolol (Lopressor)
D. Pindolol (Visken)
33. Scientific Concepts/Cardiology Explanations
Which of the following is the mechanism of action of Class III (u) A. Na+ channel blockers are Class I.
antiarrhythmic drugs? (c) B. K+ channel blockers are Class III.
Answers (u) C. Beta adrenoreceptor blockers are Class II.
A. Na+ channel blocker (u) D. Ca++ channel blockers are Class VI.
B. K+ channel blocker
C. Beta adrenoreceptor blocker
D. Ca++ channel blocker
34. Scientific Concepts/Cardiology Explanations
In congestive heart failure the mechanism responsible for the (u) A. Atrial contraction against a noncompliant ventricle is
production of an S3 gallop is the mechanism responsible for S4.
Answers (c) B. Rapid ventricular filling during early diastole is the
A. contraction of atria in late diastole against a stiffened ventricle. mechanism responsible for the S3.
B. rapid ventricular filling during early diastole. (u) C. Vibration of a partially closed mitral valve during mid
C. vibration of a partially closed mitral valve during mid to late to late diastole is the mechanism responsible for the
diastole. Austin-Flint murmur of aortic regurgitation.
D. secondary to closure of the mitral valve leaflets during systole. (u) D. Closure of the mitral valve leaflets during systole is
the mechanism responsible for part of the S1 heart sound.
35. Scientific Concepts/Cardiology Explanations
What is the most likely mechanism responsible for retinal (u) A. See B for explanation.
hemorrhages and neurologic complications in a patient with (c) B. The vegetations that occur during infective
infective endocarditis? endocarditis can become emboli and can be dispersed
Answers throughout the arterial system.
A. Metabolic acidosis (u) C. See B for explanation.
B. Systemic arterial embolization of vegetations (u) D. Glomerulonephritis and arthritis result from
C. Hypotension and tachycardia activation of the immune system.
D. Activation of the immune system
36. Scientific Concepts/Cardiology Explanations
During an inferior wall myocardial infarction the signs and symptoms (u) A. See B for explanation.
of nausea and vomiting, weakness and sinus bradycardia are a result (c) B. Increased vagal tone is common in inferior wall MI; if
of what mechanism? the SA node is involved, bradycardia may develop.
Answers (u) C. See B for explanation.
A. Increased sympathetic tone (u) D. See B for explanation.
B. Increased vagal tone
C. Activation of the renin-angiotensin system
D. Activation of the inflammatory and complement cascade system
37. Scientific Concepts/Cardiology Explanations
Which of the following is the most common cause of secondary (c) A. Renal parenchymal disease is the most common
hypertension? cause of secondary hypertension.
A. Renal parenchymal disease (u) B. Primary aldosteronism can cause secondary
B. Primary aldosteronism hypertension, but it is not the most common cause.
C. Oral contraceptive use (u) C. Oral contraceptives can cause small increases in
D. Cushing's syndrome blood pressure but considerable increases are much less
common.
(u) D. Cushing's disease is a less common cause of
secondary hypertension.
38. Clinical Therapeutics/Cardiology Explanations
Which of the following medication classes is the treatment of choice (c) A. Calcium channel blockers are effective
in a patient with variant or Prinzmetal's angina? prophylactically to treat coronary vasospasm associated
A. Calcium channel blockers with variant or Prinzmetal's angina.
B. ACE inhibitors (u) B. ACE inhibitors are not a treatment for coronary
C. Beta blockers vasospasm.
D. Angiotensin II receptor blockers (h) C. Beta blockers have been noted to exacerbate
coronary vasospasm potentially leading to worsening
ischemia.
(u) D. Angiotensin II receptor blockers are not a treatment
for coronary vasospasm.
39. Clinical Therapeutics/Cardiology Explanations
A 63 year-old female with history of diabetes mellitus presents for blood (u) A. Alpha blockers are not the treatment of choice in a
pressure follow-up. At her last two visits her blood pressure was 150/92 diabetic with hypertension.
and 152/96. Today in the office her blood pressure is 146/92. Recent (u) B. Patients with hypertension and diabetes may
blood work shows a Sodium 140 mEq/L, Potassium 4.2 mEq/L, BUN of 23 require a Beta blocker, but it should be added to an
mg/dL, and Creatinine of 1.1 mg/dL. Which of the following is the most ACE inhibitor
appropriate initial medication in this patient? if the ACE inhibitor is ineffective on its own.
A. Terazosin (Hytrin) (c) C. ACE inhibitors should be part of the initial
B. Atenolol (Tenormin) treatment of hypertension in diabetics because of
C. Lisinopril (Zestril) beneficial effects in
D. Hydrochlorothiazide (HCTZ) diabetic nephropathy and is the most appropriate initial
medication.
(u) D. Patients with hypertension and diabetes mellitus
may require a diuretic, but it should be added to an
ACE
inhibitor if the ACE inhibitor is ineffective on its own.
40. Diagnostic Studies/Cardiology Explanations
What is the EKG manifestation of cardiac end-organ damage due to (u) A. Right bundle branch block is caused by a delay in
hypertension? the conduction system in the right ventricle. It may be
A. Right bundle branch block caused by right ventricular hypertrophy or conditions
B. Left ventricular hypertrophy with higher pulmonic resistance such as cor pulmonale.
C. Right ventricular hypertrophy Hypertension, however, is likely to cause changes in the
D. ST segment elevation in lateral precordial leads left ventricle rather than the right ventricle.
(c) B. Long-standing hypertension can lead to left
ventricular hypertrophy with characteristic changes
noted on EKG.
(u)C. See A for explanation.
(u) D. ST segment elevation is a sign of acute myocardial
infarction not hypertension.
41. Health Maintenance/Cardiology Explanations
Annual blood pressure determinations should be obtained beginning at (c) A. Periodic measurements of blood pressure should
the age of be part of routine preventive health assessments
A. 3 years. beginning at the age of 3 years.
B. 5 years. (u) B. See A for explanation.
C. 12 years. (u) C. See A for explanation.
D. 18 years. (u) D. See A for explanation.
42. History & Physical/Cardiology Explanations
Which of the following conditions would cause a positive Kussmaul's (u) A. Left ventricular failure results in the back-up of
sign on physical examination? blood into the left atrium and then the pulmonary
A. Left ventricular failure system so it would not be associated with Kussmaul's
B. Pulmonary edema sign.
C. Coarctation of the aorta (u) B. Pulmonary edema primarily results in increased
D. Constrictive pericarditis pulmonary pressures rather than having effects on the
venous inflow into the heart.
(u) C. Coarctation of the aorta primarily affects outflow
from the heart due to the stenosis resulting in delayed
and decreased femoral pulses; it has no effect on
causing Kussmaul's sign.
(c) D. Kussmaul's sign is an increase rather than the
normal decrease in the CVP during inspiration. It is most
often caused by severe right-sided heart failure; it is a
frequent finding in patients with constrictive pericarditis
or right ventricular infarction.
43. History & Physical/Cardiology Explanations
Which of the following physical findings is suggestive of atrial septal defect? (c) A. An atrial septal defect will cause a shunt
A. Fixed split S2 of blood from the left to the right atrium. This
B. Increased pulse pressure will result in an equalization in the amount of
C. Continuous mechanical murmur blood entering both the left and right
D. Difference in blood pressure between the left and right arm ventricles which effectively eliminates the
normally wide splitting that inspiration
typically causes in hearts without an atrial
septal defect.
(u) B. Pulse pressures reflect the difference in
aortic and left ventricular volumes that occur
during ventricular systole Increased pulse
pressures are seen in aortic regurgitation
which is a different entity than atrial septal
defect.
(u) C. Continuous mechanical murmurs are
noted in patients with patent ductus
arteriosus.
(u) D. Differences in blood pressure between
the left and right arms are seen in conditions
such as coarctation of the
aorta.
44. Clinical Therapeutics/Cardiology Explanations
A 29 year-old male presents with complaint of substernal chest pain for 12 hours. (u) A. Pericardiocentesis is the treatment of
The patient states that the pain radiates to his shoulders and is relieved with choice in a patient with a pericardial effusion
sitting forward. The patient admits to recent upper respiratory symptoms. On and cardiac tamponade, there is no evidence
examination vital signs are BP 126/68, HR 86, RR 20, temp 100.3 degrees F. There is of either of these in this patient.
no JVD noted. Heart exam reveals regular rate and rhythm with no S3 or S4. There (u) B. Nitroglycerin is indicated in the
is a friction rub noted. Lungs are clear to auscultation. EKG shows diffuse ST treatment of chest pain related to angina.
segment elevation. What is the treatment of choice in this patient? (u) C. Percutaneous coronory intervention is
A. Pericardiocentesis the treatment of choice in a patient with an
B. Nitroglycerin acute myocardial infarction.
C. Percutaneous coronary intervention (c) D. Indomethacin, a nonsteroidal anti-
D. Indomethacin (Indocin) inflammatory medication, is the treatment of
choice in a patient with acute
pericarditis.
45. Diagnosis/Cardiology Explanations
A 24 year-old male presents for routine physical examination. On physical (u) A. Patients with hypertrophic obstructive
examination, you find that the patient's upper extremity blood pressure is higher cardiomyopathy do not present with
than the blood pressure in the lower extremity. Heart exam reveals a late systolic hypertension or weak femoral pulses.
murmur heard best posteriorly. What is the most likely diagnosis in this patient? (u) B. The murmur associated with patent
A. Hypertrophic obstructive cardiomyopathy foramen ovale is a systolic ejection murmur
B. Patent foramen ovale heard in the second and third intercostal
C. Coarctation of the aorta spaces and patients do not present with
D. Patent ductus arteriosus hypertension.
(c) C. Coarctation of the aorta commonly
presents with higher systolic pressures in the
upper extremities than the lower extremities
and absent or weak femoral pulses.
(u) D. Patent ductus arteriosus is rare in
adults and patients are noted to have a
continuous rough, machinery murmur.
46. Diagnosis/Cardiology Explanations
A 63 year-old female presents with a complaint of chest pressure for one (u) A. A patient with aortic dissection will complain of
hour, noticed upon awakening. She admits to associated nausea, tearing, ripping pain. EKG is often normal, but may
vomiting, and shortness of breath. 12 lead EKG reveals ST segment reveal left ventricular strain pattern.
elevation in leads II, III, and AVF. Which of the following is the most likely (c) B. Myocardial infarction often presents with chest
diagnosis? pressure and associated nausea and vomiting. ST
A. Aortic dissection segment elevation in leads II, III, and AVF are classic
B. Inferior wall myocardial infarction findings seen in acute inferior wall myocardial
C. Acute pericarditis infarction.
D. Pulmonary embolus (u) C. Acute pericarditis presents with atypical chest
pain and diffuse ST segment elevation.
(u) D. Pulmonary embolism often presents with either
no EKG changes or sinus tachycardia. Classically
described,
rarely seen findings include a large S wave in lead I, a
Q wave with T wave inversion in lead III, ST segment
depression in lead II, T wave inversion in leads V1-V4
and a transient right bundle branch block.
47. History & Physical/Cardiology Explanations
A 12 month-old child with tetralogy of Fallot is most likely to have which (u) A. Chest pain is not a feature of tetralogy of Fallot.
of the following clinical features? (c) B. Cyanosis is very common in tetralogy of Fallot.
A. Chest pain (u) C. Convulsions are occasionally seen as part of
B. Cyanosis severe hypoxic spells in infancy rather than a feature
C. Convulsions of tetralogy
D. Palpitations of Fallot.
(u) D. Palpitations are uncommon in tetralogy of
Fallot.
48. Diagnosis/Cardiology Explanations
A 23 year-old male presents with syncope. On physical examination you (c) A. Hypertrophic cardiomyopathy is characterized
note a medium-pitched, mid-systolic murmur that decreases with by a medium- pitched, mid-systolic murmur that
squatting and increases with straining. Which of the following is the most decreases with squatting and increases with
likely diagnosis? straining.
A. Hypertrophic cardiomyopathy (u) B. Straining decreases the intensity of the
B. Aortic stenosis murmur associated with aortic stenosis and squatting
C. Mitral regurgitation increases the intensity.
D. Pulmonic stenosis (u) C. Mitral regurgitation is characterized by a
blowing systolic murmur that radiates to the axilla, it is
not often associated with syncope.
(u) D. Pulmonic stenosis is a harsh systolic murmur
with a widely split S2, and no change with
maneuvers.
49. Health Maintenance/Cardiology Explanations
A patient with which of the following is at highest risk for (u) A. Congenital heart disease is not an established risk factor for
coronary artery disease? coronary artery disease.
A. Congenital heart disease (u) B. While patients with polycystic ovary syndrome have
B. Polycystic ovary syndrome hyperinsulimemia, they do not have the same poor
C. Acute renal failure prognosis for coronary artery disease as patients with diabetes
D. Diabetes mellitus mellitus.
(u) C. Patients with acute renal failure are not at risk for coronary
artery disease, although patients with diabetes and
chronic renal disease do have this risk.
(c) D. Patients with diabetes mellitus are in the same risk category for
coronary artery disease as those patients with
established atherosclerotic disease.
50. Clinical Therapeutics/Cardiology Explanations
Acute rebound hypertensive episodes have been reported (u) A. Verapamil is a calcium channel blocker and there is no
to occur with the sudden withdrawal of associated rebound hypertension after withdrawal.
A. verapamil (Calan). (u) B. Lisinopril is an ACE inhibitor, which is not associated with
B. lisinopril (Prinivil). rebound hypertension.
C. clonidine (Catapres). (c) C. Clonidine (Catapres) is a central alpha agonist and abrupt
D. hydrochlorothiazide (HCTZ) withdrawal may produce a rebound hypertensive
crisis.
(u) D. Hydrochlorothiazide is a thiazide diuretic, which is not
associated with rebound hypertension.
51. Diagnosis/Cardiology Explanations
A 38 year-old female with history of coarctation of the aorta repair at the age of two presents with fevers (u) A. Acute MI
for four weeks. The patient states that she has felt fatigued and achy during this time. Maximum presents with
temperature has been 102.1 degrees F. She denies cough, congestion, or other associated symptoms. complaint of chest
Physical examination reveals a pale tired appearing female in no acute distress. Heart reveals a new grade pain, SOB, not with
III-IV/VI systolic ejection border at the apex, and a II/VI diastolic murmur at the right sternal border. fever and myalgias.
What is the most likely diagnosis? (c) B. Bacterial
A. Acute myocardial infarction endocarditis
B. Bacterial endocarditis presents as febrile
C. Acute pericarditis illness lasting several
D. Restrictive cardiomyopathy days to weeks,
commonly with
nonspecific
symptoms,
echocardiogram
often reveals
vegetations on
affected valves.
(u) C. Pericarditis
does not present
with systolic or
diastolic murmur or
vegetation, more
commonly
pericardial friction
rub would be noted.
(u) D. Restrictive
cardiomyopathy will
show impaired
diastolic filling on
echocardiogram and
is not associated with
fever.
52. Diagnostic Studies/Cardiology Explanations
A 23 year-old female with history of palpitations presents for evaluation. She (u) A. Cardiac catheterization evaluates coronary
admits to acute onset of rapid heart beating lasting seconds to minutes with arteries but has no role in the diagnosis of
associated shortness of breath and chest pain. The patient states she can supraventricular tachycardia.
relieve her symptoms with valsalva. Which of the following is the most (u) B. Cardiac MRI cannot diagnose and define
appropriate diagnostic study to establish a definitive diagnosis in this patient? pathway of supraventricular tachycardia.
A. Cardiac catheterization (u) C. Chest CT scan will not establish definitive
B. Cardiac MRI diagnosis of supraventricular tachycardia.
C. Chest CT scan (c) D. Electrophysiology study is useful in
D. Electrophysiology study establishing the diagnosis and pathway of
complex arrhythmias such as
supraventricular tachycardia.
53. Clinical Therapeutics/Cardiology Explanations
Which of the following is the chief adverse effect of thiazide diuretics? (c) A. Thiazide diuretics can induce electrolyte
A. Hypokalemia changes. Principle among those is hypokalemia.
B. Hypernatremia (u) B. Hyponatremia, not hypernatremia may be
C. Hypocalcemia a complication of thiazide diuretics.
D. Hypermagnesemia (u) C. Thiazide diuretics cause the retention of
calcium and would not cause hypocalcemia.
(u) D. Thiazide diuretics cause the retention of
calcium and do not readily affect magnesium
levels.
54. Clinical Intervention/Cardiology Explanations
A 25 year-old male with history of syncope presents for evaluation. The patient (c) A. Radiofrequency catheter ablation is the
admits to intermittent episodes of rapid heart beating that resolve treatment of choice on patients with accessory
spontaneously. 12 Lead EKG shows delta waves and a short PR interval. Which pathways, such as Wolff-Parkinson-White
of the following is the treatment of choice in this patient? Syndrome.
A. Radiofrequency catheter ablation (h) B. Calcium channel blockers such as
B. Verapamil (Calan) verapamil decrease refractoriness of the
C. Percutaneous coronary intervention accessory pathway or increase that of the AV
D. Digoxin (Lanoxin) node leading to faster ventricular rates,
therefore calcium channel blockers should be
avoided in patients with WPW.
(u) C. Percutaneous coronary intervention is
indicated in the treatment of coronary artery
disease, not preexcitation syndromes.
(h) D. Digoxin decreases refractoriness of the
accessory pathway and increases that of the AV
node leading to faster ventricular rates. It should
therefore be avoided in patients with WPW.
55. History & Physical/Cardiology Explanations
A patient presents for a follow-up visit for chronic hypertension. Which of the (u) A. Cherry-red fovea and boxcar
following findings may be noted on the fundoscopic examination of this segmentation of the retinal veins are findings
patient? seen in central retinal artery occlusion.
A. cherry-red fovea (u) B. See letter A for explanation.
B. boxcar segmentation of retinal veins (u) C. Papilledema is noted in conditions causing
C. papilledema increased intracranial pressure.
D. arteriovenous nicking (c) D. Arteriovenous nicking is common in chronic
hypertension.
56. Diagnostic Studies/Cardiology Explanations
Which of the following diagnostic tests should be ordered initially to evaluate for (u) A. Venogram has been replaced by
suspected deep venous thrombosis of the leg? noninvasive tests due to discomfort, cost,
A. Venogram technical difficulties, and complications,
B. Arteriogram such as phlebitis.
C. Duplex ultrasound 56
D. Impedance plethysmography (h) B. Thrombophlebitis is a venous
problem, not an arterial one. Any
unnecessary invasive procedure is
potentially harmful.
(c) C. Ultrasound is the technique of
choice to detect deep venous thrombosis
in the leg.
(a) D. Impedance plethysmography is
equivalent to ultrasound in detecting
thrombi of the femoral and popliteal
veins,
but it may miss early, nonocclusive
thrombi.
57. Diagnosis/Cardiology Explanations
A 36 year-old patient with cardiomyopathy secondary to viral myocarditis develops (c) A. Given the presence of
fatigue, increasing dyspnea, and lower extremity edema over the past 3 days. He cardiomyopathy, the patient's heart has
denies fever. A chest x-ray shows no significant increase in heart size, but reveals decreased functional reserve. The
prominence of the superior pulmonary vessels. Based on these clinical findings, which symptoms and chest x-ray findings are
of the following is the most likely diagnosis? typical of congestive heart failure.
A. Heart failure (u) B. Endocarditis occurs as a result of
B. Subacute bacterial endocarditis infection that primarily occurs in the
C. Pulmonary embolus blood stream. Endocarditis would
D. Pneumonia present with signs of infection or seeding
rather than signs of heart failure.
(u) C. Pulmonary embolus usually
presents with an acute onset of chest
pain, severe dyspnea, and anxiety.
(u) D. Pneumonia is less likely since there
is no fever and edema is not usually
associated with pneumonia.
58. Clinical Intervention/Cardiology Explanations
Which of the following is first-line treatment for symptomatic bradyarrhythmias due (c) A. Permanent pacemakers are the
to sick sinus syndrome (SSS)? therapy of choice in patients with
A. Permanent pacemaker symptomatic bradyarrhythmias in sick
B. Radiofrequency ablation sinus syndrome.
C. Antiarrhythmics (u) B. Radiofrequency ablation is used
D. Anticoagulation therapy for the treatment of accessory pathways
in the heart. (u) C. See A for explanation.
(u) D. See A for explanation.
59. History & Physical/Cardiology Explanations
What type of chest pain is most commonly associated with a dissecting aortic aneurysm? (u) A. Squeezing pain is more
A. Squeezing characteristic of angina or
B. Dull, aching esophageal pain.
C. Ripping, tearing (u) B. Dull, aching pain is more
D. Burning characteristic of chest wall pain,
possibly angina, or anxiety.
(c) C. A dissecting aortic aneurysm
often presents with a very severe
ripping, tearing-like pain.
(u) D. Burning pain is more
characteristic of esophageal reflux,
esophagitis, or tracheobronchitis.
60. Health Maintenance/Cardiology Explanations
A 52 year-old obese female with a history of hypertension, tobacco abuse, and (c) A. Age is a non modifiable risk
hyperlipidemia presents for routine follow-up. Which of her risk factors for coronary factor, as is family history of
atherosclerosis is not modifiable? premature coronary heart disease
A. Age (u) B. High LDL is a modifiable risk
B. High LDL factor, as is Hypertension, low HDL,
C. Hypertension obesity, tobacco abuse, physical
D. Obesity inactivity
(u) C. See B for explanation.
(u) D. See B for explanation.
61. Diagnosis/Cardiology Explanations
An 8 year-old boy is brought to a health care provider complaining of dyspnea and fatigue. (u) A. Ventricular septal defect
On physical examination, a continuous machinery murmur is heard best in the second left causes a holosystolic murmur
intercostal space and is widely transmitted over the precordium. The most likely diagnosis rather than a continuous
is machinery-like murmur.
A. ventricular septal defect. (u) B. Atrial septal defect causes a
B. atrial septal defect. fixed split S2 rather than a
C. congenital aortic stenosis. continuous systolic heart murmur.
D. patent ductus arteriosus. (u) C. Congenital aortic stenosis
causes a crescendo-decrescendo
systolic murmur heard best in the
second
intercostal space.
(c) D. Patent ductus arteriosus is
classically described in children as a
continuous machinery-type
murmur that is
widely transmitted across the
precordium.
62. Diagnostic Studies/Cardiology Explanations
A 63 year-old male with history of hypertension and tobacco abuse presents (u) A. Helical CT scan aids in the diagnosis
complaining of dyspnea on exertion for two weeks. The patient admits to one of pulmonary embolism, not in the
episode of chest discomfort while shoveling snow which was relieved after five evaluation of angina.
minutes of rest. Vital signs are BP 130/70, HR 68, RR 14. Heart exam reveals regular rate (u) B. Chest x-ray is not used as a
and rhythm, normal S1 and S2, no murmur, gallop, or rub. Lungs are clear to diagnostic study to evaluate symptoms of
auscultation bilaterally. There is no edema noted. Which of the following is the most angina or coronary heart disease.
appropriate initial diagnostic study for this patient? (c) C. In patients with classic symptoms of
A. Helical CT scan angina, nuclear stress testing is the most
B. Chest x-ray widely used test for diagnosis of
C. Nuclear stress test ischemic heart disease.
D. Cardiac catheterization (u) D. Coronary angiography is indicated
in patients with classic stable angina who
are severely symptomatic despite
medical therapy and are being
considered for percutaneous
intervention (PCI), patients with
troublesome symptoms that are difficult
to diagnose, angina symptoms in a
patient who has survived sudden cardiac
death event, patients with ischemia on
noninvasive testings.
63. Clinical Intervention/Cardiology Explanations
A 52 year-old male with history of hypertension and hyperlipidemia presents with an (u) A. Percutaneous coronary
acute myocardial infarction. Urgent cardiac catheterization is performed and shows intervention is a better, less invasive
a 90% occlusion of the left anterior descending artery. The other arteries have alternative to CABG for single vessel
minimal disease. Ejection fraction is 45%. Which of the following is the treatment of coronary artery disease.
choice in this patient? (h) B. Streptokinase is not commonly used
A. Coronary artery bypass grafting (CABG) for treatment of acute myocardial
B. Streptokinase infarction because it is ineffective at
C. Percutaneous coronary intervention (PCI) opening the occluded artery and
D. Warfarin (Coumadin) reducing mortality. Streptokinase would
be harmful because it would increase the
risk of bleeding.
(c) C. Immediate coronary angiography
and primary percutaneous coronary
intervention have been shown to be
superior to thrombolysis.
(u) D. Warfarin is used to prevent
thrombosis and not for acute treatment.
64. Clinical Intervention/Cardiology Explanations
A patient presents with an acutely painful and cold left leg. Distal pulses are absent. (u) A. Vena cava filters are used in the
Leg is cyanotic. There are no signs of gangrene or other open lesions. Symptoms management of venous thromboembolic
occurred one hour ago. Which of the following treatments is most appropriate? disease when anticoagulation cannot be
A. Vena cava filter done.
B. Embolectomy (c) B. Embolectomy within 4 to 6 hours is
C. Amputation the treatment of choice.
D. Aspirin (h) C. Amputation is done only when no
viable tissue is present. Cutting off a
viable limb is never a good idea.
(u) D. Aspirin is used in the prevention
and treatment of coronary disease and
has no role in the treatment of
peripheral arterial embolism.
65. Clinical Therapeutics/Cardiology Explanations
Which of the following medications used in the treatment of (u) A. Digoxin is not used for the acute termination of
supraventricular tachycardia is able to cause sinus arrest and supraventricular tachycardia.
asystole for a few seconds while it breaks the paroxysmal (c) B. Adenosine is an endogenous nucleoside that results in
supraventricular tachycardia? profound (although transient) slowing of the AV
A. Digoxin (Lanoxin) conduction and sinus node discharge rate. This agent has a very
B. Adenosine (Adenocard) short half-life of 6 seconds.
C. Verapamil (Calan) (u) C. Although verapamil may be used for the termination of
D. Quinidine (Quinaglute) acute supraventricular tachycardia, it does not lead to
sinus arrest in therapeutic doses.
(u) D. Quinidine is rarely used today and is not indicated for the
termination of supraventricular tachycardia.
66. Diagnosis/Cardiology Explanations
An elderly female presents for evaluation of exertional (c) A. The major symptoms of aortic stenosis are exertional syncope,
syncope, dyspnea, and angina. She admits that previous to dyspnea, and angina. Symptoms do not become apparent for a
these symptoms she had insidious progression of fatigue that number of years and usually are not present until the valve is
caused her to curtail her activities. Which of the following is narrowed to less than 0.5 cm to 2 cm of valve surface area.
the most likely diagnosis? (u) B. Patients with aortic regurgitation are likely to complain of an
A. Aortic stenosis uncomfortable awareness of their heart, especially when lying
B. Aortic regurgitation down. These patients develop sinus tachycardia with exertion and
C. Mitral stenosis complain of palpitations and head pounding with activity.
D. Mitral valve prolapse (u) C. The symptoms related to mitral stenosis are related to
increased pulmonary pressure after the left atrium can no longer
overcome the outflow obstruction.
(u) D. Patients with mitral valve prolapse are typically
asymptomatic throughout their lives, although a wide range of
symptoms is possible. When symptoms do occur, palpitations from
arrhythmias are most common along with lightheadedness.
Syncope is not part of this disease process.
67. History & Physical/Cardiology Explanations
Which of the following would you expect on physical (u) A. Mitral stenosis is a diastolic, not a systolic murmur.
examination in a patient with mitral valve stenosis? (c) B. Mitral stenosis is characterized by a mid-diastolic opening
A. Systolic blowing murmur snap.
B. Opening snap (u) C. Mid-systolic clicks are noted in mitral valve prolapse, not
C. Mid-systolic click mitral stenosis.
D. Paradoxically split S2 (u) D. Paradoxical splitting of S2 occurs in aortic stenosis not mitral
stenosis.
68. Scientific Concepts/Cardiology Explanations
Which of the following is the most common cause for acute (u) A. Coronary microemboli occlusion is a rare cause of acute
myocardial infarction? myocardial infarction.
A. Occlusion caused by coronary microemboli (c) B. Acute myocardial infarction occurs when a coronary artery
B. Thrombus development at a site of vascular injury thrombus develops rapidly at a site of vascular
C. Congenital abnormalities injury. In most cases, infarction occurs when an atherosclerotic
D. Severe coronary artery spasm plaque fissures, ruptures, or ulcerates and when conditions favor
thrombogenesis, so that a mural thrombus forms at the site of
rupture and leads to coronary artery occlusion.
(u) C. Congenital abnormalities are rare causes of acute MI.
(u) D. Severe coronary artery spasm is more likely to result in
Prinzmetal's angina rather than true infarction.
69. Health Maintenance/Cardiology Explanations
A 78 year-old male with history of coronary artery disease status post CABG and ischemic cardiomyopathy (c) A. Home
presents with complaint of progressive dyspnea and orthopnea. He also complains of lower extremity monitoring of daily
edema. The patient denies fever, chest pain, or cough. On physical examination, vital signs are BP 120/68, weights can alert
HR 75 and regular, RR 22, afebrile. You note the patient to have an S3 heart sound, jugular venous the health care
distention, and 2+ lower extremity edema. The patient is admitted and treated. Upon discharge from the provider to the
hospital, the patient should be educated to monitor which of the following at home? early recognition of
A. Daily weights worsening heart
B. Daily spirometry failure.
C. Daily blood glucose (u) B. Spirometry
D. Daily fat intake monitoring is
important in a
patient with
asthma, not heart
failure.
(u) C. Daily blood
glucose monitoring
is important in a
patient with
diabetes, not heart
failure.
(u) D. Daily fat
intake is important,
but will not improve
his heart failure
management.
70. Scientific Concepts/Cardiology Explanations
Which of the following is the most common cause of arterial embolization? (u) A. Rheumatic
A. Rheumatic heart disease heart disease is a
B. Myxoma rare cause of
C. Atrial fibrillation embolization
D. Venous thrombosis (u) B. Myxoma is a
rare cause of
embolization.
(c) C. Atrial
fibrillation is
present in 60-70%
of patients with
arterial emboli and
is associated with
left atrial
appendage
thrombus.
(u) D. Venous
thrombosis may be
a cause of
embolization
paradoxically, but is
uncommon.
71. Scientific Concepts/Cardiology Explanations
The most common arrhythmia encountered in patients with mitral stenosis is (u) A. See B for explanation.
A. atrial flutter. (c) B. Mitral stenosis leads to enlargement of
B. atrial fibrillation. the left atrium, which is the major
C. paroxysmal atrial tachycardia. predisposing risk factor for the
D. atrio-ventricular dissociation. development of atrial fibrillation.
(u) C. See B for explanation.
(u) D. See B for explanation.
72. Clinical Therapeutics/Cardiology Explanations
Long term use of which of the following drugs may cause a drug-induced lupus- (u) A. Prednisone is not implicated in drug-
type eruption? induced skin reactions.
A. prednisone (u) B. Tetracycline and sulfonamides are
B. tetracycline known to cause a photosensitive rash on sun
C. procainamide exposed areas of the skin.
D. oral contraceptives (c) C. Procainamide and hydralazine are the
most common drugs that may cause a lupus-
like eruption.
(u) D. Oral contraceptives may induce
erythema nodosum.
73. Scientific Concepts/Cardiology Explanations
Which of the following is a cause of high output heart failure? (u) A. Low output heart failure occurs
A. myocardial ischemia secondary to ischemic heart disease,
B. complete heart block hypertension, dilated cardiomyopathy,
C. aortic stenosis valvular and pericardial disease, and
D. thyrotoxicosis arrhythmia.
(u) B. See A for explanation.
(u) C. See A for explanation.
(c) D. High output heart failure occurs in
patients with reduced systemic vascular
resistance. Examples include: thyrotoxicosis,
anemia, pregnancy, beriberi and Paget's
disease. Patients with high output heart
failure usually have normal pump function,
but it is not adequate to meet the high
metabolic demands.
74. Diagnosis/Cardiology Explanations
A 46 year-old male with no past medical history presents complaining of chest (c) A. Myocarditis often occurs secondary to
pain for four hours. The patient admits to feeling very poorly over the past two acute viral illness and causes cardiac
weeks with fever and upper respiratory symptoms. The patient denies shortness dysfunction. Patients will commonly have a
of breath or diaphoresis. On examination the patient appears fatigued. Vital signs history of a recent febrile illness. Chest pain
reveal a BP of 130/80, HR 90 and regular, RR 14. The patient is afebrile. Labs reveal a may mimic that of a myocardial infarction and
Troponin I of 10.33 ug/L (0-0.4ug/L). Cardiac catheterization shows normal Troponin I levels maybe elevated in one-
coronary arteries and an ejection fraction of 40% with global hypokinesis. Which third of patients. Contractile dysfunction is
of the following is the most likely diagnosis? seen on catheterization and/or
A. myocarditis echocardiogram.
B. pericarditis (u) B. Pericarditis does not typically cause
C. hypertrophic cardiomyopathy ventricular dysfunction and cardiac enzymes
D. coronary artery disease are usually normal.
(u) C. Hypertrophic cardiomyopathy is
associated with ventricular hypercontractility.
(u) D. This patient had normal coronary
arteries on cardiac catheterization, no signs of
coronary artery disease.
75. Clinical Therapeutics/Cardiology Explanations
Which of the following antihypertensive agents is considered to have both (c) A. Carvedilol has both alpha- and beta-
alpha- and beta-blocker activities? blocker activities.
A. carvedilol (Coreg) (u) B. Hydralazine and minoxidil are
B. hydralazine (Apresoline) considered vasodilators.
C. minoxidil (Loniten) (u) C. See B for explanation.
D. spironolactone (Aldactone) (u) D. Spironolactone is a potassium-sparing
diuretic.
76. Diagnosis/Cardiology Explanations
A 12 year-old boy presents to the office with pain in his legs with activity (u) A. Abdominal aortic aneurysm is usually
gradually becoming worse over the past month. He is unable to ride a bicycle asymptomatic until the patient has dissection
with his friends due to the pain in his legs. Examination of the heart reveals an or rupture. It is uncommon in a child.
ejection click and accentuation of the second heart sound. Femoral pulses are (u) B. Pheochromocytoma classically causes
weak and delayed compared to the brachial pulses. Blood pressure obtained in paroxysms of hypertension due to
both arms is elevated. Chest x-ray reveals rib notching. Which of the following is catecholamine release from the adrenal
the most likely diagnosis? medulla, but does not cause variations in blood
A. abdominal aortic aneurysm pressure in the upper and lower extremities.
B. pheochromocytoma (c) C. Coarctation is a discrete or long segment
C. coarctation of the aorta of narrowing adjacent to the left subclavian
D. thoracic outlet syndrome artery. As a result of the coarctation, systemic
collaterals develop. X-ray findings occur from
the dilated and pulsatile intercostal arteries
and the "3" is due to the coarctation site with
proximal and distal dilations.
(u) D. Thoracic outlet syndrome occurs when
the brachial plexus, subclavian artery, or
subclavian vein becomes compressed in the
region of the thoracic outlet. It is the most
common cause of acute arterial occlusion in the
upper extremity of adults under 40 years old.
77. Clinical Therapeutics/Cardiology Explanations
According to the recent JNC VII guidelines, a 34 year-old male who has type 1 (u) A. Beta blockers could potentially be
diabetes mellitus and hypertension should be started on which type of harmful in a patient with diabetes mellitus. Use
antihypertensive agent? a cardioselective beta- blocker to reduce the
A. beta-blocker incidence of hypoglycemia.
B. loop diuretic (u) B. See C for explanation.
C. ACE inhibitor (c) C. ACE inhibitors are effective in young
D. thiazide diuretic patients. They are capable of providing
protection to the kidney especially in diabetes
mellitus.
(u) D. See C for explanation.
78. Scientific Concepts/Cardiology Explanations
A patient presents with moderate mitral stenosis. Which of the (c) A. 50-80% of patients with mitral stenosis will develop
following complications is associated with an increased risk of paroxysmal or chronic atrial fibrillation; 20-30% of patients with
systemic embolization in this patient? atrial fibrillation will have systemic embolization.
A. atrial fibrillation (u) B. Pulmonary hypertension can occur in patients with severe
B. pulmonary hypertension mitral stenosis with symptoms of low cardiac output and right
C. increased left atrial pressure sided heart failure. Pulmonary hypertension does not cause
D. left ventricular dilatation systemic embolization.
(u) C. Patients with mitral stenosis can have increased left atrial
pressures relative to the left ventricular diastolic pressures; this
does not usually cause systemic embolization.
(u) D. Left ventricular dilatation is more common in aortic valve
disease than mitral valve disease.
79. Diagnostic Studies/Cardiology Explanations
A 19 year-old female presents with complaint of palpitations. (u) A. Patients with Marfan's syndrome commonly have mitral
On examination you note the patient to have particularly long valve prolapse and possibly aortic regurgitation. Right atrial
arms and fingers and a pectus excavatum. She has a history of enlargement, pulmonic stenosis and ventricular septal defect are
joint dislocation and a recent ophthalmologic examination not commonly seen.
revealed ectopic lentis. Which of the following echocardiogram (c) B. This patient has the signs and symptoms consistent with
findings would be most consistent with this patient's physical Marfan's syndrome. Ectopia lentis, aortic root dilation and aortic
features? dissection are major criteria for the diagnosis of the disease.
A. right atrial enlargement B. aortic root dilation (u) C. See A for explanation. (u) D. See A for explanation.
C. pulmonic stenosis
D. ventricular septal defect
80. Diagnosis/Cardiology Explanations
A patient presents with chest pain. ECG done in the emergency (u) A. Anterior wall myocardial infarction is characterized by ST
department reveals ST segment elevation in leads II, III, and segment elevation in 1 or more of the precordial (V1- V6) leads.
AVF. This is most consistent with a myocardial infarction in (c) B. Inferior wall myocardial infarction is characterized by ST
which of the following areas? segment elevation in leads II, III, and AVF.
A. anterior wall (u) C. Posterior wall myocardial infarction is characterized by ST
B. inferior wall segment depression in leads V1-V3 and a large R wave in leads
C. posterior wall V1-V3.
D. lateral wall (u) D. Lateral wall myocardial infarction is characterized by ST
segment elevation in leads I and AVL.
81. Clinical Therapeutics/Cardiology Explanations
Which of the following is an absolute contraindication to (u) A. See C for explanation.
thrombolytic therapy in a patient with an acute ST segment (u) B. See C for explanation.
elevation myocardial infarction? (c) C. Absolute contraindications to thrombolytic therapy include
A. history of severe hypertension presently controlled B. a previous hemorrhagic stroke, a stroke within one year, a
current use of anticoagulation therapy known intracranial neoplasm, active internal bleeding, and a
C. previous hemorrhagic stroke suspected aortic dissection. Severe, but controlled hypertension,
D. active peptic ulcer disease use of anticoagulation, and active peptic ulcer disease are relative
contraindications in which the risk/benefit ratio must be weighed
in each patient.
(u) D. See C for explanation.
82. Health Maintenance/Cardiology Explanations
A postmenopausal woman is at greatest risk of death from (u) A. See B for explanation.
which of the following? (c) B. Although women tend to be concerned about dying from
A. stroke breast cancer, heart disease is the number one killer of
B. heart disease postmenopausal women.
C. ovarian cancer (u) C. See B for explanation.
D. breast cancer (u) D. See B for explanation.
83. Diagnosis/Cardiology Explanations
A 46 year-old female is being evaluated for a new-onset hypertension that was (u) A. Pheochromocytoma will result in an
discovered on screening at her workplace. The patient had several readings increase in the production and release of
revealing systolic and diastolic hypertension. Patient is currently on no catecholamines, which results in an increase in
medications. Physical examination is unremarkable. A complete laboratory urinary metanephrines on testing.
evaluation revealed hypokalemia as the only abnormality. Which of the (u) B. Renal artery stenosis is identified by an
following is the most likely diagnosis for this patient? abnormal radionuclide uptake on the affected
A. pheochromocytoma kidney.
B. renal artery stenosis (u) C. Coarctation of the aorta is identified by
C. coarctation of the aorta delayed and weakened femoral pulses along
D. primary aldosteronism with a blood pressure in the lower extremities
significantly lower than in the upper extremities.
(c) D. Primary aldosteronism has an increased
aldosterone secretion, which causes the
retention of sodium and the loss of potassium.
This should be the primary consideration for this
patient.
84. Clinical Intervention/Cardiology Explanations
A 54 year-old female who has diabetes presents with rubor, absence of hair, (u) A. Vasodilator therapy is not indicated.
and brittle nails of her left foot. She complains of leg pain that awakens her at (c) B. Bypass surgery is indicated in the presence
night. Examination reveals a femoral bruit with diminished popliteal and of rest pain and provides relief of symptoms in
pedal pulses on the left side. The most appropriate therapy would be 80 to 90% of patients.
A. vasodilator therapy. (u) C. While an exercise program is appropriate
B. bypass surgery. with claudication, rest pain is a surgical
C. exercise program. indication.
D. embolectomy. (u) D. Embolectomy is used for acute arterial
occlusion.
85. Clinical Therapeutics/Cardiology Explanations
Which electrolyte abnormality is associated with an increase in the risk for (u) A. See B for explanation.
digoxin toxicity?] (c) B. Decreased concentration of potassium
A. hypercalcemia results in the increased activity of cardiac
B. hypokalemia glycosides by increasing tissue binding and
C. hypermagnesemia decreasing renal excretion of digoxin. Potassium
D. hyponatremia loss is the only significant electrolyte abnormality
that significantly affects digoxin metabolism.
(u) C. See B for explanation.
(u) D. See B for explanation.
86. Health Maintenance/Cardiology Explanations
A 56 year-old male, status post myocardial infarction, is noted to have left (u) A. See D for explanation.
ventricular hypertrophy and an ejection fraction of 38%. Which of the (u) B. See D for explanation.
following medications should be prescribed to prevent the development of (u) C. See D for explanation.
heart failure symptoms? (c) D. ACE inhibitors have been shown to
A. amlodipine (Norvasc) markedly improve survival and are also
B. furosemide (Lasix) recommended for prevention of symptoms in
C. hydrochlorothiazide (HCTZ) patients at risk for heart failure.
D. lisinopril (Zestril)
87. Health Maintenance/Cardiology Explanations
A 74 year-old patient presents with signs and symptoms of heart failure. EKG shows the (u) A. Calcium channel blockers are
patient to be in atrial fibrillation at a rate of 80 bpm. Blood pressure is 120/76. The utilized in rate control of atrial
patient denies complaint of palpitations, chest pain, or syncope. Which of the fibrillation. This patient's rate is
following is the most important long term therapy in this patient? controlled at 80bpm presently.
A. verapamil (Calan) (u) B. Antiarrhythmic therapy may be
B. amiodarone (Cordarone) considered in patients with atrial
C. furosemide (Lasix) fibrillation; however anticoagulation
D. warfarin (Coumadin) therapy must occur first.
(u) C. Diuretics may be indicated in the
acute treatment of heart failure;
however they may not be needed long
term.
(c) D. Patients with atrial fibrillation have
an increased risk for stroke, therefore
these patients need anticoagulation
with warfarin to an INR of 2.0-3.0.
88. Diagnostic Studies/Cardiology Explanations
Which of the following ECG findings is consistent with hyperkalemia? (u) A. Prolonged QT interval is seen in
A. prolonged QT interval hypocalcemia.
B. delta wave (u) B. Delta wave is a sign of ventricular
C. peaked T waves preexcitation seen in Wolf-Parkinson-
D. prominent U waves White (WPW) Syndrome.
(c) C. Narrowing and peaking of T
waves are the beginning EKG changes
associated with hyperkalemia.
(u) D. Prominent U waves are a sign of
prolonged ventricular repolarization
seen in hypokalemia.
89. History & Physical/Cardiology Explanations
A 58 year-old male presents with chest pain. Vital signs include blood pressure of (c) A. Malignant hypertension is
210/175, pulse 80, RR 20. Which of the following would you expect to find on physical characterized by marked blood
examination? pressure elevation with papilledema,
A. papilledema often with encephalopathy or
B. carotid bruit nephropathy.
C. diastolic murmur (u) B. Carotid bruits are associated with
D. absent peripheral pulses carotid artery stenosis.
(u) C. Diastolic murmurs are associated
with valvular heart disease such as
aortic regurgitation and mitral stenosis.
(u) D. Peripheral pulses are absent in
acute arterial occlusion or severe
peripheral arterial disease.
90. Clinical Therapeutics/Cardiology Explanations
A 55 year-old diabetic female presents for a 3 month blood pressure follow- (u) A. This patient's blood pressure is controlled;
up. At the last visit the BP was 160/90 for the third consecutive visit. She was there is no indication at this time to add an
placed on an ACE inhibitor and educated regarding lifestyle modifications. At additional drug.
today's visit the patient complains of persistent annoying dry cough that has (u) B. This patient's chronic dry cough is likely
been going on since the last visit. BP today is 120/70. What is the best secondary to the ACE inhibitor, the medication
recommendation to control her BP? should be stopped, however the patient needs
A. add a diuretic something for blood pressure control.
B. stop the ACE inhibitor and continue lifestyle modifications (c) C. This patient's chronic dry cough is likely
C. switch patient to an Angiotensin II Receptor Blocker (ARB) secondary to the ACE inhibitor, the medication
D. do nothing and recheck BP in 3 months should be stopped. Angiotensin II Receptor
Blockers (ARBs) are similar to ACE inhibitors for BP
control, but do not cause cough.
(u) D. This patient's chronic dry cough is likely
secondary to the ACE inhibitor, the medication
should be stopped to encourage compliance.
91. Diagnosis/Cardiology Explanations
A newborn is seen for an initial two week visit. Physical examination reveals a (c) A. Patent ductus arteriosus is characterized by
thrill and a continuous machinery murmur in the left second intercostal a classic harsh, machinery-like murmur that is
space. Which of the following is the most likely diagnosis? continuous through systole and diastole. This is
A. patent ductus arteriosus heard best at the left second interspace and is
B. ventricular septal defect commonly associated with a thrill.
C. tetralogy of Fallot (u) B. Ventricular septal defect is characterized by
D. coarctation of the aorta a holosystolic murmur at the lower left sternal
border.
(u) C. Tetralogy of Fallot is characterized by a
systolic thrill at the left sternal border with a
systolic ejection murmur that may or may not have
an associated systolic click.
(u) D. Coarctation of the aorta is associated with a
systolic ejection click or a short systolic murmur at
the left sternal border.
92. History & Physical/Cardiology Explanations
A patient had an acute inferior, transmural myocardial infarction 4 days ago. (u) A. This is a classic description of mitral stenosis.
A new murmur raises the suspicion of mitral regurgitation due to papillary (u) B. This is a classic description for pulmonic
muscle rupture. Which of the following murmur descriptions describes this stenosis.
condition? (u) C. This is a classic description for mitral valve
A. A grade III/VI diastolic murmur heard best at the apex without radiation. prolapse.
B. A grade IV/VI systolic ejection murmur heard best at the base with (c) D. This is a classic description of mitral
radiation to the left clavicle. regurgitation. The papillary muscle rupture is a
C. A grade II/VI systolic murmur heard best at the apex preceded by a click complication of an acute inferior transmural
and without radiation. myocardial infarction, and results in a failure of
D. A grade IV/VI systolic murmur heard best at the apex with radiation to the the mitral valve leaflets to close. The direction of
left axilla. regurgitant flow of blood is toward the left axilla.
93. Clinical Intervention/Cardiology Explanations
A 58 year-old male who is otherwise healthy presents with chest pain and is (u) A. See D for explanation.
found to have left main coronary artery stenosis of 75%. The most important (u) B. See D for explanation.
aspect of his management now is (u) C. See D for explanation.
A. daily aspirin to prevent MI. (c) D. Although medical therapy is important,
B. nitrate therapy for the angina. revascularization is indicated when stenosis of the
C. aggressive risk factor reduction. left main coronary artery is greater than 50%.
D. referral for coronary artery revascularization.
94. Diagnostic Studies/Cardiology Explanations
A 17 year-old woman presents to the office with recurrent episodes of (u) A. A cardiac catheterization will not be
palpitations and near syncope. Initial ECG was normal. She is concerned about useful since the patient is at low risk for actual
these episodes since they can occur at any time. Which of the following is the coronary artery disease.
most appropriate step to pursue in her evaluation? (u) B. Tilt table testing is useful only in trying to
A. cardiac catheterization determine vasodepressor syncope that is
B. tilt table testing related to position.
C. echocardiogram (u) C. An echocardiogram shows valves and left
D. Holter monitoring ventricle function, not pathways of conduction.
(c) D. Holter monitoring will identify the heart
rhythm; an event recorder may also be useful
in this setting if the Holter monitor is not
diagnostic.
95. Health Maintenance/Cardiology Explanations
A 37 year-old female with history of Turner's syndrome and coarctation of the (c) A. This patient has a history of congenital
aorta repaired at the age of 3 presents for routine examination. The patient is heart disease and presently has a murmur
without complaints of chest pain, dyspnea, palpitations, or syncope. On consistent with aortic regurgitation. This
examination vitals signs reveal a BP of 130/76, HR 70, regular, RR 16. On cardiac patient requires antibiotic prophylaxis against
examination you note a grade II/VI systolic ejection murmur at the left sternal infective endocarditis.
border and a grade III/VI blowing diastolic murmur. Which of the following does (u) B. Beta blocker therapy may increase the
this patient require? amount of regurgitation because of increased
A. antibiotic prophylaxis diastolic time and is not indicated in this
B. beta blocker therapy patient.
C. chest CT (u) C. This patient should undergo serial
D. exercise stress test echocardiograms, chest CT will not give
information regarding any changes in the
aortic regurgitation or ejection fraction.
(u) D. This patient is without any complaints;
exercise stress test is not indicated.
96. Diagnostic Studies/Cardiology Explanations
A 60 year-old male with history of hypertension and hyperlipidemia presents (u) A. Coronary angiography is indicated in
with intermittent chest heaviness for one month. The patient states he has had patients with classic stable angina who are
occasional heaviness in his chest while walking on his treadmill at home or severely symptomatic despite medical therapy
shoveling snow. He also admits to mild dyspnea on exertion. His symptoms are and are being considered for percutaneous
relieved with 2-3 minutes of rest. He denies lightheadedness, syncope, intervention (PCI), patients with troublesome
orthopnea or lower extremity edema. Vitals reveal a BP of 130/90, HR 70, regular, symptoms that are difficult to diagnose, angina
RR 14. Cardiac examination revealed a normal S1 and S2, without murmur or symptoms in a patient who has survived
rub. Lungs were clear to auscultation. Extremities are without edema. EKG sudden cardiac death event, patients with
reveals no acute change and cardiac enzymes are negative. Which of the ischemia on noninvasive testings, a stress test is
following is the most appropriate next diagnostic study? a better initial diagnostic study for this patient.
A. cardiac catheterization (c) B. This patient has signs and symptoms of
B. nuclear exercise stress test classic angina; nuclear stress testing is the most
C. helical (spiral) CT useful noninvasive procedure for diagnosis of
D. transthoracic Echocardiogram ischemic heart disease and evaluation of
angina in this patient.
(u) C. Helical CT is used in the diagnosis of
pulmonary embolism, not in the evaluation of
angina.
(u) D. Echocardiogram is used in the evaluation
of valvular heart disease not in the evaluation
of suspected myocardial ischemia.
97. History & Physical/Cardiology Explanations
A 28 year-old patient presents with complaint of chest pain for two days. The patient (u) A. Lower extremity edema is
describes the pain as constant and sharp. It is worse with lying down, better with sitting up seen with heart failure or
and leaning forward. Vital signs are BP 120/80, HR 80, regular, RR 14 and Temperature 100.1 venous insufficiency, not
degrees F. Which of the following would you expect to find on physical examination? pericarditis.
A. lower extremity edema (u) B. Carotid bruits are
B. carotid bruit associated with carotid artery
C. pericardial friction rub stenosis, not pericarditis.
D. splinter hemorrhages (c) C. This patient has signs and
symptoms of pericarditis. A
pericardial friction rub is
characteristic of acute
pericarditis.
(u) D. Subungual (splinter)
hemorrhages are characteristic
of infective endocarditis, not
pericarditis.
98. History & Physical/Cardiology Explanations
A 22 year-old male received a stab wound in the chest an hour ago. The diagnosis of pericardial (u) A. Pulmonary edema may
tamponade is strongly supported by the presence of result with low output states as
A. pulmonary edema. seen with myocardial
B. wide pulse pressure. contusions, but it is not strongly
C. distended neck veins. suggestive of tamponade.
D. an early diastolic murmur. (u) B. Wide pulse pressure is
seen in conditions of high stroke
volume such as aortic
insufficiency or
hyperthyroidism. Narrow pulse
pressure is seen with cardiac
tamponade.
(c) C. Cardiac tamponade will
manifest with distended neck
veins and cold clammy skin.
(u) D. The onset of diastolic
murmur is suggestive of valvular
disease, not tamponade.
99. Clinical Intervention/Cardiology Explanations
A 45 year-old female presents with complaint of lower extremity discomfort. The patient (u) A. Heparin is used in the
admits to dull aching of the left lower extremity. The discomfort is worse after standing for treatment of deep vein
long periods of time. Examination reveals dilated, tortuous and elongated veins on the medial thrombosis not varicose veins.
aspect of the left leg. Pedal pulses are +2/4 bilaterally. There are no skin changes or lower (c) B. This patient has signs and
extremity edema noted. Which of the following is the most appropriate initial treatment of symptoms of varicose veins.
choice in this patient? Initial treatment with
A. heparin compression stockings may
B. compression stockings prolong or avoid the need for
C. furosemide (Lasix) surgery.
D. thrombectomy (u) C. This patient has no signs of
edema or venous insufficiency
requiring diuretic therapy.
(u) D. Thrombectomy is
indicated in a patient with an
arterial thrombus, this patient
has intact pulses and no pallor.
100. Diagnosis/Cardiology Explanations
A 55 year-old male presents with complaint of sudden ripping chest (u) A. Pain associated with a myocardial infarction is
pain that radiates into the abdomen. On examination the patient is commonly a retrosternal pressure, squeezing, or
found to have diminished peripheral pulses and a diastolic murmur. heaviness. ST segment elevation on EKG would be
EKG reveals left ventricular hypertrophy. Which of the following is expected.
the most likely diagnosis? (u) B. A pulmonary embolism is associated with
A. acute myocardial infarction retrosternal pain; however chest pain is not always
B. pulmonary embolism present. Patients more commonly will have a sudden onset
C. acute pericarditis of dyspnea. PE is not usually associated with a diastolic
D. aortic dissection murmur or diminished pulses.
(u) C. Acute pericarditis is characterized by sharp, knife-
like pain that is worse with lying supine and better with
sitting up and leaning forward. One would expect to find a
pericardial friction rub on auscultation.
(c) D. Aortic dissection is characterized by a ripping or
tearing type pain with radiation to the neck, back or
abdomen. Left ventricular hypertrophy is often seen on
EKG secondary to longstanding hypertension. A diastolic
murmur is often present secondary to aortic insufficiency.
101. History & Physical/Cardiology Explanations
A 78 year-old patient who is in acute distress with near-syncope and (c) A. The patient is in a third-degree heart block with the
lightheadedness is being examined. The patient's pulse is 40/min and atria contracting against a closed atrioventricular valve,
blood pressure is 90/56 mm Hg. Examination of the patient at 45 which would be the scenario in a patient who has an
degrees of elevation reveals cannon "a" waves. Which of the escape rate of 40. Elderly patients are at risk for heart
following is the most likely explanation for these abnormal waves? conduction problems such as complete heart block.
A. atrioventricular dissociation (u) B. Left ventricular hypertrophy, systolic hypertension,
B. aortic stenosis and aortic stenosis are not causes of "a" waves.
C. systolic hypertension (u) C. See B for explanation.
D. left ventricular hypertrophy (u) D. See B for explanation.
102. Scientific Concepts/Cardiology Explanations
Which of the following occurs as a result of pulmonary (u) A. See D for explanation.
hypertension? (u) B. See D for explanation.
A. left atrial enlargement (u) C. See D for explanation.
B. aortic stenosis (c) D. In pulmonary hypertension increased resistance
C. coronary artery spasm within pulmonary circulation causes the right ventricle to
D. right ventricular enlargement work harder and eventually enlarge in response. Other
changes that may occur are right atrial enlargement,
decreased left ventricular cavity size, and tricuspid
regurgitation.
103. Diagnostic Studies/Cardiology Explanations
A 56 year-old male presents to the office with a history of abdominal (u) A. Although some abdominal aortic aneurysms are
aortic aneurysm. He was told that he will need ongoing evaluation to calcified, abdominal radiography may demonstrate the
assess whether the aneurysm is expanding. What is the calcified outline of the aneurysm. However, about 25% of
recommended study to utilize in this situation? aneurysms are not calcified and cannot be visualized by
A. plain film of the abdomen plain x-ray.
B. serial abdominal exam (u) B. Serial abdominal exams are not sensitive in
C. ultrasound of the abdomen detecting progression of abdominal aortic aneurysms.
D. angiography of the abdominal aorta (c) C. An abdominal ultrasound can delineate the
transverse and longitudinal dimensions of an abdominal
aortic aneurysm and may detect mural thrombus.
Abdominal ultrasound is best used to screen patients at
risk for the development of this condition.
(u) D. Contrast aortography is used commonly for the
evaluation of patients with aneurysms before surgery,
but it has no role in the serial assessment of patients being
followed on a chronic basis.
104. Diagnosis/Cardiology Explanations
A 55 year-old morbidly obese male is seen in the office for routine (u) A. Mitral insufficiency results in the accumulation of
examination. He has a history of pulmonary hypertension and cor blood primarily in the pulmonary system and not the
pulmonale. Examination reveals a visible jugular venous pulse and a right side of the heart.
systolic flow murmur on the right side of the sternum. Which of the (c) B. Tricuspid insufficiency will result in blood being put
following is the most likely diagnosis? back into the right side of the body with increased jugular
A. mitral insufficiency pulsation in the neck, along with a palpable venous pulse
B. tricuspid insufficiency in the liver.
C. hepatic vein thrombosis (u) C. Hepatic vein thrombosis or Budd-Chiari syndrome
D. aneurysm of the thoracic aorta is associated with cirrhosis and liver clotting abnormalities
and is not due to right-sided heart failure.
(u) D. Thoracic aorta aneurysm results in a widened
mediastinum that is fairly asymptomatic until it results in
rupture or dissection. These are typically found as
incidental findings unless they are symptomatic from
dissection or rupture, which causes severe chest pain or a
severe tearing sensation in the chest.
105. Clinical Intervention/Cardiology Explanations
A 68 year-old patient presents after a syncopal episode. The patient (c) A. Patients with symptomatic ventricular tachycardia
has a history of coronary artery disease and ischemic (VT) or sustained VT and left ventricular dysfunction are at
cardiomyopathy. Echocardiogram shows an ejection fraction of 20%. increased risk for sudden cardiac death. An implantable
Electrophysiology study reveals inducible sustained ventricular defibrillator is the treatment of choice.
tachycardia from the left ventricle. Which of the following is the (u) B. Beta blockers are used in patients with
most appropriate therapy in this patient? nonsustained VT and normal ventricular function. They
A. implantable defibrillator may be used as an adjunct to, but not in place of,
B. metoprolol (Lopressor) implantable defibrillator therapy in patients with
C. radiofrequency ablation symptomatic VT or sustained VT.
D. warfarin (Coumadin) (u) C. Radiofrequency ablation is indicated in patients
with outflow tract or fascicular tachycardia, not left sided
VT.
(u) D. Anticoagulation therapy is indicated in patients with
atrial fibrillation not VT.
106. Scientific Concepts/Cardiology Explanations
Which of the following is the most common cause of infective endocarditis in an IV (u) A. See C for explanation.
drug abuser? (u) B. See C for explanation.
A. Haemophilus parainfluenza (c) C. S. aureus accounts for more than
B. Enterococci 60% of all cases of endocarditis in IV
C. Staphylococcus aureus drug abusers.
D. Viridans streptococci (u) D. See C for explanation.
107. History & Physical/Cardiology Explanations
Which of the following is an expected finding in a patient with a diagnosis of an arterial (u) A. Lower extremity edema is
embolism? commonly associated with venous
A. lower extremity edema insufficiency, not arterial embolism.
B. stasis dermatitis (u) B. Stasis dermatitis is commonly
C. palpable cord seen in patients with venous
D. pulselessness insufficiency, not arterial embolism.
(u) C. A palpable cord is commonly
found in a patient with a DVT, not
arterial embolism.
(c) D. Pulselessness is a sign of acute
ischemia secondary to arterial
embolism.
108. Scientific Concepts/Cardiology Explanations
Which of the following factors in patients with chronic venous insufficiency (u) A. Decreased intravascular oncotic
predisposes them to development of skin ulcers? pressure can cause swelling
A. Increased intravascular oncotic pressure (c) B. Leakage of fibrinogen and growth
B. Leakage of fibrinogen and growth factors into the interstitial space factors into the interstitial space,
C. Decreased capillary leakage leukocyte aggregation and activation,
D. Inherited deficiency of protein C and obliteration of the cutaneous
lymphatic network can predispose a
patient to skin ulcers
(u) C. Increased capillary leakage
causes venous insufficiency.
(u) D. Inherited deficiency of protein C
predisposes patients to thrombosis.
109. Diagnosis/Cardiology Explanations
A newborn is being evaluated for perioral cyanosis while feeding associated with (u) A. Although the murmur may be
sweating. Vital signs are rectal temperature, 37.8 degrees C (100 degrees F), blood consistent with an ASD with pulmonary
pressure 80/45 mmHg, pulse 180/min, and respirations 40/min. A grade 3/6 harsh hypertension the chest x-ray would
systolic ejection murmur with a single loud S2 is heard at the left upper sternal border. not show decreased pulmonary
Electrocardiogram (ECG) shows right ventricular hypertrophy with right axis vascular markings. With a large left to
deviation. Chest x-ray shows a boot- shaped heart and decreased pulmonary vascular right shunt large pulmonary arteries
markings. Which of the following is the most likely diagnosis? and increased vascularity would be
A. Atrial septal defect seen.
B. Total anomalous pulmonary venous return (u) B. The murmur for TAPVR is a soft
C. Coarctation of the aorta systolic murmur at the left upper
D. Tetralogy of Fallot sternal border with a split S2 in
addition to a short mid-diastolic
murmur at the low left sternal border.
(u) C. Cyanosis is usually not the
presenting sign for coarctation of the
aorta. Infants may present with heart
failure, ECG will show evidence of LVH.
(c) D. This is a common presentation for
tetralogy of fallot.
110. Clinical Therapeutics/Cardiology Explanations
A hospitalized patient is found with confirmed pulseless ventricular tachycardia. IV access is (u) A. Antiarrhythmics are
obtained following the second shock given. Which of the following medications is to be given after the third shock
administered immediately? and epinephrine has been
A. Amiodarone administered.
B. Magnesium (u) B. Magnesium is useful
C. Atropine for torsades de pointes.
D. Epinephrine (u) C. Atropine may be used
for asystole or a slow
pulseless electrical activity
(PEA) rate.
(c) D. Epinephrine should be
given as soon as IV access is
obtained before or after the
second shock.
111. Diagnosis/Cardiology Explanations
An electrocardiogram (ECG) shows a sinus rhythm with varying T wave heights, axis changes (u) A. Artifact could show a
every other beat and a wandering baseline. Which of the following is most likely the diagnosis? wandering baseline, but not
A. Artifact the distinct axis changes.
B. Digoxin toxicity (u) B. Digoxin toxicity can
C. Pericardial effusion cause bidirectional
D. Poor lead placement tachycardia, but not
electrical alternans.
(c) C. This ECG pattern best
represents pericardial
effusion due to a swinging
heart in fluid and is known
as electrical alternans.
(u) D. Poor lead placement
would show different
patterns per the leads.
112. Clinical Therapeutics/Cardiology Explanations
A 25 year-old female presents to the emergency department after an episode of substernal chest (u) A. Lorazepam is not
pain with radiation to the middle of her back that came on suddenly and lasted for about four contraindicated and can
minutes this morning while in bed. She denies previous episodes. Examination is unremarkable, help with agitation,
but she appears jittery. Toxicology screen is positive for cocaine. Which of the following psychosis or seizures.
medications is contraindicated in this patient? (u) B. Diltiazem is not
A. Lorazepam (Ativan) contraindicated but does
B. Diltiazem (Cardizem) not have a definitive role in
C. Nitroglycerin (Nitrostat) treating cocaine toxicity.
D. Propanolol (Inderal) (u) C. Nitroglycerin is not
contraindicated but does
not have a definitive role in
treating cocaine toxicity.
(c) D. Pure Beta blockers,
such as propranol, can
cause a paradoxical
hypertension because of
unopposed alpha-
adrenergic effects.
113. Clinical Intervention/Cardiology Explanations
A 56 year-old female four days post myocardial infarction presents with a new murmur. On (u) A. Although part of the
examination the murmur is a grade 3/6 pansystolic murmur radiating to the axilla. She is primary treatment to reduce
dyspenic at rest and has rales throughout all her lung fields. Blood pressure is 108/68 mmHg, mitral regurgitation, it is not
pulse 70 bpm. Which of the following would be the definitive clinical intervention? definitive.
A. Intra-aortic balloon counterpulsation (c) B. MVR is the definitive
B. Mitral valve replacement intervention to correct MR
C. Coronary artery bypass surgery caused by papillary muscle
D. Immediate fluid bolus rupture.
(u) C. CABG may be necessary
if significant blockage is found,
but it will not correct the mitral
regurgitation.
(u) D. A fluid bolus is indicated
if the patient is hypotensive.
114. Clinical Therapeutics/Cardiology Explanations
A 16 year-old male with a history of tetralogy of Fallot presents to clinic for a follow-up visit (u) A. See C for explanation.
status post replacement of his right ventricle to pulmonary artery conduit. He has complaints (u) B. See C for explanation.
of chest pain with inspiration, fever and general malaise. Cardiac examination reveals a rub (c) C. Indomethacin is suitable
with muffled heart sounds. Labs show an elevated erythrocyte sedimentation rate (ESR) and for controlling pain in
leukocytosis. Which of the following is the most effective treatment? Dressler's syndrome. ASA is
A. Acetaminophen/oxycodone preferred. Narcotics, diuretics
B. Azithromycin or antibiotics are not
C. Indomethacin recommended.
D. Furosemide (u) D. See C for explanation.
115. Diagnostic Studies/Cardiology Explanations
A 72 year-old male presents to the emergency department with crushing chest pain, dyspnea (u) A. Q waves in leads I, aVL,
and palpitations for 2 hours in duration. Enzymes are pending and he has been given aspirin V5-V6 represent infarction
and sublingual nitroglycerin. He is rushed to the catheterization lab where they find a totally involving the circumflex
occluded distal right coronary artery. Which of the following electrocardiogram (ECG) findings artery.
supports the diagnosis? (c) B. ST segment elevation in
A. Q waves in leads I, aVL, V5-V6 leads II, III, aVF, represents an
B. ST segment elevation in leads II, III, aVF acute process in the right
C. Hyperacute T waves in leads I, aVL coronary artery.
D. Flipped T waves with repolarization changes in leads V1-V4 (u) C. Hyperacute T waves in
leads I, aVL can represent the
initial changes of an infarction
involving the circumflex
artery.
(u) D. Flipped T waves with
repolarization changes in
leads V1-V4 can represent
early stages of infarction
involving the left anterior
descending artery.
116. Health Maintenance/Cardiology Explanations
Which of the following population groups represent the greatest risk for (u) A. White non-Hispanic adults have a low risk of
developing primary hypertension? hypertension compared to Hispanic and Black non-
A. White non-Hispanic Hispanics.
B. Hispanic (u) B. Hispanic adults are lower risk of hypertension
C. Mediterranean than Black non-Hispanic, but not compared to White
D. Black non-Hispanic non- Hispanic.
(u) C. Mediterranean adults have a lower risk of
hypertension than Black non-Hispanics.
(c) D. Black non-Hispanic adults have the highest risk
of hypertension.
117. Diagnosis/Cardiology Explanations
A 60 year-old male with hypertension is brought to the emergency (u) A. Although included as part of the differential
department 30 minutes after the sudden onset of severe chest pain that the presentation is not consistent with AMI. ECG may
radiates to his back and arms. His blood pressure is 180/80 mmHg in his show ST changes and a murmur of mitral
left arm; no blood pressure reading can be obtained from the right arm. regurgitation may be present with papillary muscle
ECG shows sinus tachycardia with left ventricular hypertrophy. A high rupture.
pitched decrescendo diastolic murmur is heard along the left mid-sternal (c) B. This is a classic presentation for aortic
border. Which of the following is the most likely diagnosis? dissection.
A. Acute myocardial infarction (u) C. Patients will also present with shortness of
B. Aortic dissection breath, feelings of impending doom and chest pain
C. Pulmonary embolism that varies with respirations.
D. Right subclavian arterial embolus (u) D. Arterial embolus will present with symptoms
related to the location of the occlusion. Pain and
paresthesias are usually the earliest symptoms.
118. History & Physical/Cardiology Explanations
Which of the following conditions is most suggestive of an abdominal (c) A. An abdominal aortic aneurysm presents with a
aortic aneurysm? pulsatile upper abdominal mass.
A. Abdominal mass (u) B. Hypertension is not suggestive of an
B. Hypertension abdominal aortic aneurysm.
C. Chest pain (u) C. Abdominal aortic aneurysm presents with
D. Syncope midabdominal or lower back pain.
(u) D. Syncope is not common in abdominal aortic
aneurysm, unless it ruptures.
119. Health Maintenance/Cardiology Explanations
Who is most likely to require subacute bacterial endocarditis (SBE) (u) A. See B for explanation.
prophylaxis prior to a dental procedure? (c) B. The AHA recommends that patients with
A. 22 year-old female with mitral valve prolapse prosthetic heart valves receive antibiotic
B. 36 year-old male with a bio-prosthesic mitral valve prophylaxis. As should cardiac transplant recipients
C. 45 year-old female with an ASD closure 8 months ago with no residual with valve disease, unrepaired cyanotic CHD,
defect repaired CHD with prosthetic material or device
D. 15 year-old male with a bicuspid aortic valve during the first six months of the procedure and
repaired CHD with residual defects at site of patch or
prosthetic device.
(u) C. See B for explanation.
(u) D. See B for explanation.
120. Scientific Concepts/Cardiology Explanations
When evaluating jugular venous pulsations a prominent a wave represents (c) A. The a wave corresponds to right atrial
which of the following? contraction.
A. Atrial contraction against a closed tricuspid valve (u) B. See A for explanation.
B. Rapid filling of the right atrium (u) C. See A for explanation.
C. Tricuspid regurgitation (u) D. See A for explanation.
D. Poor left ventricle compliance
121. History & Physical/Cardiology Explanations
An 18 year-old male high school basketball player comes to clinic for a routine (c) A. This murmur is most consistent with aortic
physical exam. His height is 193 cm (76 in.); arm span is 201 cm (79 in.). He has regurgitation which can be present in patients
long fingers and toes. Blood pressure is 146/62 mmHg and pulse is 64/min. with Marfans syndrome and a dilated aortic
Which of the following exam findings is most consistent with the diagnosis? root.
A. Grade 2/6 high-frequency diastolic murmur at the third right intercostal (u) B. This murmur is most consistent with an
space atrial septal defect.
B. Grade 2/6 systolic ejection murmur at the second left intercostal space with (u) C. This murmur is most consistent with a
a fixed widely split S2 patent ductus arteriosus and unlikely in this
C. Grade 2/6 continuous murmur heard best at the high left sternal border age patient.
D. Grade 2/6 systolic murmur at the fourth left intercostal space that decreases (u) D. This murmur is most consistent with
with squatting hypertrophic obstructive cardiomyopathy.
122. Scientific Concepts/Cardiology Explanations
Which of the following is the most likely initial effect on the left ventricle from (u) A. Dilitation of the ventricle is a later finding.
aortic stenosis? (u) B. This is not an effect from aortic stenosis
A. Dilitation of the ventricle with diastolic dysfunction but coronary artery blockage.
B. Wall stiffness due to ischemia from decreased coronary blood flow (u) C. Preload is the end-diastolic pressure or
C. Paradoxical wall motion abnormalities due to increased preload volume within the ventricle, ischemic heart
D. Concentric hypertrophy with preserved function disease or infarct would cause paradoxical wall
motion abnormalities due to the increased
preload.
(c) D. Hypertrophy would be the initial changes
of the left ventricle as a response to the
increased pressure.
123. Diagnosis/Cardiology Explanations
A 65 year-old female who recently had an anterior MI returns to clinic for (u) A. RV infarction is present in one-third of
follow-up six weeks after. She has no chest pain, but reports decreased exercise patients who had an inferior wall MI and
tolerance. Electrocardiogram (ECG) shows persistent ST elevation in leads V2- typically show ST elevation in V4 along with the
V4. Which of the following is the most likely diagnosis? inferior leads.
A. Right ventricular infarction (u) B. Occlusion of the right coronary artery
B. Re-occlusion of the right coronary artery would show ST elevation in the inferior ECG
C. Pericarditis leads.
D. Ventricular aneurysm (u) C. Pericarditis shows ST elevation in multiple
leads with a history consistent of a viral illness
or fever.
(c) D. Persistent ST elevation in the leads where
a previous or recent anterior MI occurred is
most likely due to ventricular aneurysm.
124. Diagnostic Studies/Cardiology Explanations
A 15 year-old male is brought to the emergency department 1 hour after an episode of (u) A. Tilt table testing should be
syncope while running in a 400-meter race. He had a similar episode 2 years ago. His performed after structural heart
mother and maternal first cousin died suddenly at the ages of 32 and 17 years, respectively. disease has been ruled out.
Examination shows abrasions of the face, hands, and knees. Neurologic examination (u) B. CT scanning of the head
shows no abnormalities. Which of the following is the most appropriate initial test? would not be the initial test of
A. Tilt table test choice.
B. CT scan of the head (c) C. ECG is the most appropriate
C. Electrocardiogram to rule out any underlying rhythm
D. Electroencephalography abnormalities, Holter or event
monitoring would also be included.
(u) D. EEG testing routinely would
not be helpful as an initial test in
this patient.
125. Clinical Therapeutics/Cardiology Explanations
What is the optimal INR for a patient with a mechanical mitral valve prosthesis on (u) A. See C for explanation.
warfarin (Coumadin)? (u) B. See C for explanation.
A. 1.5-2.5 (c) C. Patients with mechanical MV
B. 2.0-3.0 prostheses should maintain an INR
C. 2.5-3.5 between 2.5-3.5. Aortic mechanical
D. 3.5-4.5 valves can be maintained at an INR
of 2.0-3.0.
(u) D. See C for explanation.
126. Clinical Intervention/Cardiology Explanations
Which of the following can optimize quality of life and is an definitive treatment for a (u) A. Ventricular assist devices can
patient with refractory heart failure? help to provide temporary
A. Ventricular assist device circulatory support to those
B. Intra-aortic balloon counterpulsation awaiting transplantation.
C. Cardiac transplantation (u) B. IABC is useful in cardiogenic
D. Partial resection of the left ventricle shock, but does not improve
quality of life.
(c) C. Cardiac transplantation is
effective, with survival rates of 80-
90% in 1 year, 60-70% over 5 years.
It does improve quality of life
despite the immunosuppression
medications.
(u) D. Partial resection of the left
ventricle is still experimental and
has not shown to have successful
results.
127. History & Physical/Cardiology Explanations
Which of the following physical exam findings suggests worsening or severe aortic (u) A. This finding is typical in mild
stenosis? to moderate AS.
A. An ejection click preceding the murmur (u) B. This can be a normal finding.
B. A split S2 with respiration variation (u) C. This is the typical murmur for
C. Grade 2/6 systolic murmur radiating to the carotids aortic stenosis, but does not
D. Palpable thrill over the right second intercostal space suggest the degree of severity by
the grade.
(c) D. A palpable thrill or LV heave
with associated murmur suggests
severe AS.
128. Scientific Concepts/Cardiology Explanations
Perfusion of the coronary arteries occurs primarily during which of the following? (u) A. See B for explanation.
A. Systole (c) B. Coronary artery
B. Diastole perfusion occurs primarily
C. Afterload during diastole.
D. Preload (u) C. See B for explanation.
(u) D. See B for
explanation.
129. Clinical Therapeutics/Cardiology Explanations
What is the mechanism of action for aspirin? (c) A. Aspirin inhibits
A. Inhibits platelet aggregation by blocking cyclooxygenase-1 activity platelet aggregation by
B. Exerts antiplatelet effects by blocking the platelet membrane adenosine diphosphate receptors blocking cyclooxygenase-1
C. Inhibits the platelet membrane glycoprotein IIb/IIIa receptor activity.
D. Converts plasminogen to the natural fibrinolytic agent plasmin (u) B. This the mechanism
of action for ADP
antagonists such as
clopidogrel or ticlopidine.
(u) C. This is the mechanism
of action for glycoprotein
IIb/IIIa receptors such as
abciximab (ReoPro).
(u) D. This is the
mechanism of action for
tissue plasminogen
activators (tPA).
130. History & Physical/Cardiology Explanations
A 56 year-old male with a 30 pack-year smoking history presents with substernal chest pain. The (u) A. Precipitated by
pain is described as a pressure that radiates to his jaw. The pain has lasted consistently for 30 exercise and relief with rest
minutes with variable relief. His current medications include atorvastatin (Lipitor) and glyburide is consistent with angina.
(Micronase). Which of the following aggravating or relieving factors about the pain would support (c) B. Patient is having a
the diagnosis? myocardial infarction which
A. Precipitated by exercise and relieved with rest is unrelieved by rest or
B. Unrelieved by nitroglycerin nitroglycerin. (u) C.
C. Aggravated by deep breaths Aggravated by deep
D. Relieved with food breaths suggests
pericarditis.
(u) D. Relieved with food
suggests peptic ulcer
disease.
131. Health Maintenance/Cardiology Explanations
A 26 year-old female presents to clinic with a left arm that is swollen and non-tender with bluish (c) A. This is indicative of
discoloration along the upper arm and forearm. She is status post pacemaker insertion in the left migratory ecchymosis and
upper chest for third degree heart block, one week ago. Pulses are present and the arm is warm, expected after insertion of
but not red. The pacemaker incision is healing well despite a hematoma and tenderness at the a pacemaker.
site. Which of the following statements would be appropriate patient education about this (u) B. Warm compresses will
condition? help to decrease the
A. Reassurance that the discoloration is an expected finding. hematoma.
B. Apply cold compresses to the site of the hematoma. (u) C. Elevation will help to
C. Elevation of the involved extremity will increase the swelling. decrease the swelling.
D. Aspirin should be taken to help manage pain. (u) D. ASA is not the pain
medicine of choice in a
patient with a hematoma
due to its non-clotting
properties.
132. Clinical Intervention/Cardiology Explanations
A 66 year-old female with a history of coronary artery disease presents with a new onset of (c) A. This patient has
dizziness and fatigue for two weeks. She recalls nearly passing out on one occasion. Examination symptomatic second degree
is unremarkable except for bradycardia. Electrocardiogram (ECG) reveals a heart rate of 50 with a type II heart block and
normal PR interval followed by a normal QRS. There are several non-conducting P waves and no requires a pacemaker.
lengthening of the PR interval. Which of the following interventions is the therapy of choice? (u) B. Radio-frequency
A. Permanent pacemaker ablation is useful for
B. Radio-frequency ablation supraventricular
C. Maze procedure tachyarrhythmias.
D. Automatic Implantable Cardioverter Defibrillator (u) C. Maze procedure is a
surgical procedure for
decreasing the incidence of
atrial fibrillation by creating
cuts simulating a pathway in
the atriums.
(u) D. Automatic
Implantable Cardioverter
Defibrillator (AICD) is used
in patients at risk for
sudden death.
133. Diagnostic Studies/Cardiology Explanations
Pulmonary capillary wedge pressure indirectly measures which of the following? (u) A. See D for explanation.
A. Right ventricular end-diastolic pressure (u) B. See D for explanation.
B. Right atrial filling pressure (u) C. See D for explanation.
C. Left ventricular systolic pressure (c) D. Pulmonary capillary
D. Left atrial filling pressure wedge pressure indirectly
measures left atrial filling
pressures.
134. History & Physical/Cardiology Explanations
A 26 year-old male presents with increased dyspnea with exercise. He has noted a decrease in his (c) A. Hypertrophic
exercise tolerance over the past several months. He denies chest pain or skipped heart beats. cardiomyopathy can be
Echocardiogram reveals left ventricular hypertrophy with asymmetric septal hypertrophy. genetic and present in 25%
Ejection fraction is 65%. Which of the following is the most likely presenting history or physical of first degree relatives.
exam finding? (u) B. Dilated
A. He has an older brother with the same diagnosis. cardiomyopathy may
B. An S3 gallop is heard. present with an S3 gallop
C. Patient notes completing a course of adriamycin. due to volume overload.
D. Elevated jugular venous distension is noted. (u) C. Adriamycin
chemotherapy can lead to
cardiac dysfunction, dilated
cardiomyopathy and
eventually heart failure.
(u) D. Restrictive or dilated
cardiomyopathy may
present with JVD due to
abnormalities of filling.
135. Health Maintenance/Cardiology Explanations
Which of the following lifestyle recommendations would most benefit the heart (c) A. Exercise training improves activity
failure patient's quality of life? tolerance and deconditioning offering
A. Begin a regular exercise program some recompensation.
B. Total salt restriction (u) B. Although salt restriction is a
C. Home monitoring of blood pressure recommendation total salt restriction
D. Increase home oxygen requirements would be unrealistic to achieve.
(u) C. Home monitoring of blood pressure
will not improve a heart failure patient's
quality of life.
(u) D. Increasing the requirements for
home oxygen could signal that they are
worsening and is palliative for their quality
of life.
136. History & Physical/Cardiology Explanations
A newborn presents with blue discoloration of the peripheral extremities which (u) A. Raynauds phenomenon has a
worsens with exposure to cold and improves with warming. Which of the following triphasic color response and is
is the most likely cause? exacerbated by cold or emotions.
A. Raynauds phenomenon (u) B. Livedo reticularis is characterized by
B. Livedo reticularis a lacy pattern on the skin of the lower
C. Erythromelagia extremities.
D. Acrocyanosis (u) C. Erythromelagia is red, painful
extremities.
(c) D. Acrocyanosis is a blue discoloration of
the digits, usually of the peripheral
extremities in newborns, which worsens
with exposure to cold and improves with
warming.
137. Diagnosis/Cardiology Explanations
A two month-old infant appeared well until three weeks ago when he became (u) A. An ASD usually presents with a soft
dyspneic and had difficulty feeding. A 4/6 holosystolic murmur is heard at the left mid-systolic murmur in the second left ICS
lower sternal border in the 3rd ICS. An electrocardiogram (ECG) shows left and right with a widely split and fixed S2.
ventricular hypertrophy. Which of the following is the most likely diagnosis? (u) B. With pulmonary hypertension the
A. Atrial septal defect murmur may be most consistent with
B. Pulmonary hypertension pulmonary or tricuspid insufficiency.
C. Ventricular septal defect (c) C. This is a classic presentation for a
D. Tricuspid insufficiency ventricular septal defect.
(u) D. Tricuspid insufficiency is a systolic
murmur heard best at the 4th ICS that
may radiate to the apex.
138. Diagnostic Studies/Cardiology Explanations
Which of the following laboratory tests would be elevated in a patient with (u) A. LDH is not a sensitive marker in
symptomatic heart failure? patients with heart failure.
A. Lactate dehydrogenase (LDH) (u) B. Troponin measurements are specific
B. Troponin I (TnI) for myocardial infarctions.
C. C-reactive protein (CRP) (u) C. Increased serum levels of CRP are
D. Brain natriuretic peptide (BNP) found in patients with unstable angina
and MI. They can be a strong predictor of
coronary events.
(c) D. BNP is expressed in the ventricles
and is a sensitive assay in patients with
heart failure.
139. Clinical Therapeutics/Cardiology Explanations
When prescribing loop diuretics which of the following electrolytes should be most closely (u) A. See B for
monitored? explanation.
A. Calcium (c) B. Potassium along
B. Potassium with magnesium should
C. Sodium be monitored when
D. Chloride prescribing loop
diuretics.
(u) C. See B for
explanation.
(u) D. See B for
explanation.
140. Diagnostic Studies/Cardiology Explanations
Which of the following studies is the best initial diagnostic evaluation to estimate ventricular size (u) A. ECG is not
and hypertrophy? sensitive or reliable to
A. Electrocardiogram (ECG) estimate ventricular size
B. Cardiac CT scan and hypertrophy.
C. Echocardiogram (u) B. Cardiac CT scan
D. Myocardial perfusion imaging can detect coronary
calcification, but is most
sensitive to assess
disorders of the aorta.
(c) C. Echocardiogram
provides the safest and
most reliable means to
evaluate ventricular
size, hypertrophy and
function.
(u) D. Myocardial
perfusion imaging is
used for measurement
of LV ejection fraction
and assess regional wall
motion abnormalities.
141. Diagnosis/Cardiology Explanations
A 44 year-old female presents to clinic for evaluation of a syncopal episode that occurred while (u) A. This patient would
walking her dog two days ago. She denies amnesia or head trauma. She has had increasing dyspnea more likely have right
on exertion and pedal edema. Physical examination reveals clubbing of her fingers and central heart failure than left
cyanosis. Auscultation of the heart reveals tricuspid insufficiency, widely split second heart sound heart failure.
with a palpable P2. Echocardiogram reveals a large ostium secundum atrial septal defect with (u) B. Ebsteins anomaly
bidirectional flow. Which of the following is a secondary complication in this patient? is apical displacement of
A. Left heart failure the septal tricuspid
B. Ebsteins anomaly leaflet and not caused
C. Tricuspid stenosis by an ASD.
D. Pulmonary hypertension (u) C. Tricuspid stenosis
is not caused by an ASD.
(c) D. Her symptoms
and exam findings are
consistent with
pulmonary
hypertension and in
her case, Eisenmengers
disease, which is a late
finding.
142. Clinical Intervention/Cardiology Explanations
Which of the following treatments will most benefit the diabetic (u) A. See D for explanation.
patient with two vessel coronary disease? (u) B. See D for explanation.
A. Stent placement (u) C. See D for explanation.
B. Percutaneous balloon angioplasty (c) D. CABG is the treatment of choice in a diabetic with
C. Medical management two or three vessel disease.
D. Coronary artery bypass graft
143. Diagnosis/Cardiology Explanations
A 24 year-old male comes to the clinic with a one week history of pain (c) A. Axillary-subclavian venous thrombosis can occur
and swelling that involves the entire right upper extremity. He exercises in someone who strenuously exercises, has had a
frequently and has noticed the pain worsening while lifting weights. central venous catheter or history of venous
Examination shows enlarged cutaneous veins over the right anterior thrombosis.
chest wall with a palpable cord. His right hand appears dusky. Which of (u) B. Thromboangiitis obliterans involves the arteries,
the following is the most likely diagnosis? although the smaller veins can be included and is
A. Axillary-subclavian venous thrombosis linked to tobacco use.
B. Thromboangiitis obliterans (u) C. This presentation is not consistent with
C. Superficial thrombophlebitis of the cephalic vein superficial thrombophlebitis and there is no history of
D. Brachial artery occlusion varicosities or previous IVs.
(u) D. Symptoms are not consistent with brachial
arterial occlusion.
144. History & Physical/Cardiology Explanations
When performing a pre-participation sports physical in the adolescent (c) A. Hypertrophic cardiomyopathy (HCM) is a known
population, a murmur with which of the following qualities indicates a cause of sudden death during or just after physical
risk for sudden death during exercise? exertion and competitive sports. The murmur
A. Increases with the Valsalva maneuver associated with HCM is worsened by conditions that
B. Increases with squatting maneuver cause reduced ventricular volume such as the Valsalva
C. Associated with a mid-systolic click maneuver, sudden standing, and tachycardia.
D. Mid-systolic without radiation to the carotids (u) B. Typical systolic flow murmurs will become
accentuated with maneuvers which increase venous
blood flow to the heart and these murmurs do not
place athletes at risk for sudden cardiac death.
(u) C. Mitral valve prolapse is the most common type of
heart murmur that is associated with a mid-systolic
click. Mitral valve prolapse does not place the patient at
risk for sudden cardiac death.
(u) D. A mid-systolic heart murmur that fails to radiate
into the carotids is most commonly associated with a
benign systolic flow murmur and does not place the
athlete at risk for sudden cardiac death.
145. Clinical Therapeutics/Cardiology Explanations
Which of the following antiarrhythmic drugs can be associated with (u) A. See B for explanation.
hyper- or hypothyroidism following long-term use? (c) B. Amiodarone is structurally related to thyroxine
A. Quinidine and contains iodine, which can induce a hyper- or
B. Amiodarone hypothyroid state.
C. Digoxin (u) C. See B for explanation.
D. Verapamil (u) D. See B for explanation.
146. Diagnosis/Cardiology Explanations
A 56 year-old male with a (u) A. See B for explanation.
known history of (c) B.
polycythemia suddenly Arterialthrombosishasoccurredandisevidencedbythelossofthepoplitealanddorsalispedispulse.Thisis
complains of pain and a surgical emergency. Venous occlusion and thrombophlebitis do not result in loss of arterial pulse.
paresthesia in the left leg. (u) C. See B for explanation.
Physical examination reveals (u) D. See B for explanation.
the left leg to be cool to the
touch and the toes to be
cyanotic. The popliteal pulse
is absent by palpation and
Doppler. The femoral pulse is
absent by palpation but weak
with Doppler. The right leg
and upper extremities have
2+/4+ pulses throughout.
Given these findings what is
the most likely diagnosis?
A. Venous thrombosis
B. Arterialthrombosis
C. Thromboangiitis obliterans
D. Thrombophlebitis
147. History & Physical/Cardiology Explanations
A 25 year-old female presents (u) A. Pulsus paradoxus is a classic finding for cardiac tamponade.
with a three day history of (u) B. Localized crackles are associated with pneumonia and consolidation, not pericarditis.
chest pain aggravated by (c) C. Pericardial friction rub is characteristic of an inflammatory pericarditis.
coughing and relieved by (u) D. Wheezing is characteristic for pulmonary disorders, such as asthma.
sitting and leaning forward.
She is febrile and a CBC with
differential reveals
leukocytosis. Which of the
following physical
examination signs is
characteristic of her problem?
A. Pulsus paradoxus
B. Localized crackles
C. Pericardial friction rub
D. Wheezing
148. Diagnosis/Cardiology Explanations
During physical examination (u) A. Arterial ulcers typically are the last in the sequence of events of peripheral arterial disease;
an elderly patient is noted to which include decreased or absent pulses distal to the blockage, muscle atrophy, hair loss,
have a painless, brown- thickened nails, smooth and shiny skin, reduced skin temperature, pallor, cyanosis, ulcers, and
colored ulceration in the area gangrene. Arterial ulcers typically occur on the feet in the areas of pressure points.
of the medial malleolus. (c) B. Venous ulceration develops in the lower extremity secondary to venous incompetence and
Which of the following is the chronic edema. The medial aspect of the ankle is the most common location.
most likely diagnosis? (u) C. Arterial insufficiency is most likely to present with symptoms of claudication prior to the
A. Arterial ulcer development of skin ulcers. When these ulcers do occur, they are most commonly seen as arterial
B. Venous ulcer ulcers and not venous ulcers.
C. Arterial insufficiency (u) D. Diabetic ulcers typically occur due to atherosclerosis and arterial insufficiency along with
D. Diabetic ulcer diabetic peripheral neuropathy. These ulcers are more likely to occur on pressure points on the foot
and fail to heal because of poor circulation to these areas.
149. Health Maintenance/Cardiology Explanations
Which of the following is a proven risk factor for the (u) A. Infective endocarditis is not associated with the development of
development of abdominal aortic aneurysm? abdominal aortic aneurysm.
A. Infective endocarditis (u) B. Diabetic patients do have a higher rate of atherosclerosis, but there
B. Diabetes mellitus is no clear causal evidence of diabetics being at higher risk for the
C. Cigarette smoking development of abdominal aortic aneurysm.
D. Alcohol abuse (c) C. Cigarette smoking is the primary risk factor for the development of
aortic aneurysms.
(u) D. Alcohol abuse is not related to development of abdominal aortic
aneurysm.
150. Clinical Therapeutics/Cardiology Explanations
When instituting diuretic therapy for patients with (u) A. Thiazide diuretics may have better hypertension control than the
heart failure, which of the following is considered the short acting loop diuretics but they are generally ineffective when the
treatment of choice as first-line therapy in a failing glomerular filtration rate falls below 30-40 mL/min.
kidney due to its improved sodium clearance? (c) B. Loop diuretics remain active in severe renal insufficiency and are the
A. Hydrochlorothiazide (Diuril) most effective type of diuretics used in the management of heart failure
B. Bumetanide (Bumex) symptoms. These agents have a rapid onset and result in natriuresis due
C. Spironolactone (Aldactone) to their activity in the ascending limb in the Loop of Henle.
D. Acetazolamide (Diamox) (u) C. Potassium-sparing diuretics have very weak diuretic properties and
are useful as adjunctive therapy in patients with Stage 3 or Stage 4 heart
failure who are already on a first-line diuretic and other agents.
(u) D. Acetazolamide is a carbonic anhydrase inhibitor and works by
causing a metabolic acidosis with loss of bicarbonate in the failing kidney.
It is a weak diuretic and does not work in the setting of a failing kidney.
151. Diagnostic Studies/Cardiology Explanations
High ventricular filling pressures are indicated by a (c) A. Brain natriuretic peptide (BNP) is a hormone released from the
rise in which of the following? myocardium when stretched such as with high ventricular filling
A. Brain natriuretic peptide pressures from CHF. It can indicate or be used as prognostic evaluation in
B. Troponin patients with acute CHF.
C. Myoglobin (u) B. Troponin is a marker for cardiac muscle damage as in an acute
D. Creatinine myocardial infarction or ischemia.
(u) C. Myoglobin is a byproduct of muscle destruction or damage from
ischemia or other causes of skeletal muscle damage. Myoglobin may rise
with high ventricular pressures, however it is nonspecific.
(u) D. Creatinine is a marker of renal function.
152. Clinical Intervention/Cardiology Explanations
Which of the following is first-line treatment for a (c) A. Permanent pacemakers are the therapy of choice in patients with
symptomatic bradyarrhythmia due to sick sinus symptomatic bradyarrhythmias in sick sinus syndrome.
syndrome? (u) B. Radiofrequency ablation is used for the treatment of accessory
A. Permanent pacemaker pathways in the heart.
B. Radiofrequencyablation (u) C. See A for explanation.
C. Antiarrhythmic therapy (u) D. See A for explanation.
D. Anticoagulation therapy
153. Scientific Concepts/Cardiology Explanations
What is the most likely mechanism responsible for (u) A. See B for explanation.
retinal hemorrhages and neurologic complications in (c) B. The vegetations that occur during infective endocarditis can become
a patient with infective endocarditis? emboli and can be dispersed throughout the arterial system.
A. Metabolicacidosis (u) C. See B for explanation.
B. Embolizationofvegetations (u) D. Glomerulonephritis and arthritis result from activation of the
C. Hypotension and tachycardia immune system.
D. Activation of the immune system
154. Diagnostic Studies/Cardiology Explanations
Which of the following is a non-invasive (c) A. The single most useful index is the ankle pressure. This can be obtained with an
quick method of evaluating a patient with Ankle-Brachial Index (ABI) which the severity of signs and symptoms of arterial
suspected lower extremity arterial insufficiency are correlated to the findings on the ABI. It is a non-invasive study that
insufficiency? can be performed in an office setting.
A. Ankle-Brachial Index (u) B. Striker Tonometry is used to evaluate compartment pressures.
B. Striker Tonometry (u) C. CT angiography is an invasive test involving radiation and contrast that cannot
C. CT Angiography be done in an office setting.
D. Lower extremity arteriography (u) D. Lower extremity arteriography is an invasive test involving radiation and
contrast that cannot be done in an office setting.
155. History & Physical/Cardiology Explanations
Which of the following is the earliest (u) A. Right ventricular failure is manifested by dependent edema and congestion in
symptom for patients with left ventricular the lungs.
failure? (c) B. Patients with left ventricular heart failure may be comfortable at rest and may
A. Dependentedema experience their first symptoms with dyspnea with conversation or with mild exertion.
B. Dyspneaonexertion (u) C. Right ventricular failure is manifested by congestion in the lungs with cough
C. Congestion being a possible manifestation of this congestion.
D. Chest pain (u) D. Patients who have on-going left ventricular failure may have underlying
coronary heart disease. When the heart failure is progressing, chest pain (manifested
by angina) and myocardial infarction may be manifestations of ongoing, progressive
heart failure. Acute decline in heart failure may lead to acute myocardial infarction but
this tends to be a very late symptom.
156. Clinical Therapeutics/Cardiology Explanations
When utilizing medical treatment for (u) A. Beta blockers primarily work by decreasing cardiac contraction and slowing the
hypertension, which of the following heart rate. There is no association with hyperkalemia.
classes of medications should be used with (u) B. Calcium channel blockers (especially the dihydropyridines) act as vasodilators
caution in those with elevated potassium with some effect at lessening cardiac contraction. Their use is not associated with
levels or intrinsic renal disease? hyperkalemia.
A. Beta blockers (u) C. Central alpha agonists stimulate the alpha receptors in the brain resulting in
B. Calcium channel blockers decreased vessel wall pressures. This process is responsible for causing a lowering of
C. Central alpha agonists the blood pressure. There is little effect on the kidney and these agents do not cause
D. ACE inhibitors hyperkalemia.
(c) D. ACE inhibitors are recognized as valuable agents in the management of
hypertension but they may result in hyperkalemia in patients with intrinsic renal
disease. They should be used with caution in patients who are taking potassium-
sparing diuretics. These agents reduce glomerular filtration pressure resulting in
retention of potassium since less filtration occurs in the kidney.
157. Clinical Intervention/Cardiology Explanations
A 60 year-old male has unstable angina, but (u) A. Thrombolysis is recommended in acute embolic occlusion, not chronic.
is otherwise healthy. A 90% lesion is found (u) B. Medical management is appropriate only for patients who are not surgical
in the left main coronary artery. Which of candidates.
the following interventions is most (c) C. CABG is indicated in patients with stenosis of the left main coronary artery and
appropriate? those with three-vessel coronary artery disease.
A. Thrombolysis with t-PA (u) D. Percutaneous transluminal coronary angioplasty is not the management of
B. Medical management with nitrates choice in left mainstem artery disease because of increased potential complications
C. Coronary artery bypass graft (CABG) and mortality.
D. Percutaneous transluminal coronary
angioplasty
158. Scientific Concepts/Cardiology Explanations
Which of the following pathogens has been linked with the (u) A. Human papilloma virus most commonly is associated
development of acute myocarditis? with venereal warts and not myocarditis. (u) B. Rotavirus is
A. Human papilloma virus primarily responsible for acute diarrhea.
B. Rotavirus (u) C. Human Herpes Virus 6 is the causative organism for
C. Human Herpes Virus 6 Roseola.
D. Coxsackie B virus (c) D. Although associated with a number of infectious and
systemic diseases, myocarditis is most frequently the result of
a viral infection, with Coxsackie B virus and echovirus being
the most frequently implicated in the infection.
159. Diagnosis/Cardiology Explanations
A patient is having a routine physical examination. Funduscopic (c) A. Hypertensive retinopathy may cause AV narrowing and
examination reveals AV narrowing and venous nicking. The nasal venous nicking due to these blood vessels having increased
border of the optic disc appears blurred. Which of the following is pressures.
the most likely underlying cause? (u) B. Macular degeneration is associated with the formation
A. Hypertension of Drusen and neoproliferation.
B. Maculardegeneration (u) C. Retinal detachment is observed funduscopically by the
C. Retinal detachment retina being displaced from its attachment. Patients will
D. Diabetes mellitus present with complaints of floaters in the eye field or abrupt
loss of vision if the detachment is complete.
(u) D. Diabetes mellitus is most closely associated with
neovascularization and microaneurysms as its primary
manifestations.
160. History & Physical/Cardiology Explanations
A 33 year-old female presents to the office with a complaint of (u) A. Atrial fibrillation, the most common cause of sustained
palpitations. There is no history of any significant heart disease in irregular heart rates, tends to present in an older population
the past and her symptoms begin and end abruptly on their own. or in patients that have underlying heart disease such as
Which of the following is the most likely explanation for these mitral stenosis.
symptoms? (u) B. Atrial flutter is not commonly seen in young patients
A. Atrialfibrillation without underlying heart disease although it does tend to be
B. Atrialflutter intermittent in nature when it does occur.
C. Benign supraventricular tachycardia (c) C. Benign supraventricular tachycardia tends to occur in a
D. Sinus tachycardia young patient without pre-existing heart disease. Symptoms
begin and end abruptly without therapy and occurrences
happen only intermittently. It is the most likely diagnosis in
this setting.
(u) D. Sinus tachycardia may occur in a young patient
without underlying heart disease but the palpitations and
tachycardia tends to start and stop gradually rather than
abruptly.
161. Diagnosis/Cardiology Explanations
A patient's EKG reveals widened P waves in lead II and large (u) A. Right atrial enlargement is found on EKG with peaked P
negative deflection of the P wave in lead V1. Which of the waves in lead II and a large positive deflection on the initial P
following is the most likely underlying cause for this? wave in lead V1.
A. Right atrial enlargement (c) B. Wide P waves in lead II and a deep negative deflection
B. Left atrial enlargement in lead V1 is due to P-mitrale which is caused by left atrial
C. Right ventricular hypertrophy enlargement.
D. Left ventricular hypertrophy (u) C. Right ventricular hypertrophy is noted on the EKG by
having a large R wave in lead AvR along with a deep S wave
in leads V5 or V6.
(u) D. On an EKG, left ventricular hypertrophy is associated
with tall R waves in leads V5 and V6, deep S waves in AvR and
V1, and tall R waves in AvL and AvF.
162. Health Maintenance/Cardiology Explanations
According to the American College of Cardiology/American (u) A. According to the ACC/AHA 2005 guidelines, patients with risk
Heart Association classification of heart failure, which of factors for heart disease but who have yet to develop symptoms are
the following patients fits the Stage B Classification system? categorized as Stage A. These patients have hypertension and lipid
A. Asymptomatic patient with no structural disease or disorders treated along with lifestyle modifications.
patients who are at high risk for the development of heart (c) B. According to the ACC/AHA 2005 guidelines, patients with
failure. structural heart disease who have not yet experienced symptoms
B. Asymptomatic patient with structural heart disease. are classified as Stage B. This is the initial stage in which medication
C. Symptomatic patient with structural heart disease. therapy other than just ACE inhibitors are recommended.
D. Patients with refractory symptoms despite intervention. (u) C. According to the ACC/AHA 2005 guidelines, patients with
symptoms and structural heart disease are classified as Stage C.
(u) D. According to the ACC/AHA 2005 guidelines, patients with
refractory symptoms are classified as Stage D.
163. Diagnosis/Cardiology Explanations
A 15 year-old male presents acutely to the office. His legs are (u) A. Pheochromocytoma is most commonly associated with
cool to the touch. Examination reveals that his pulses and palpitations and feelings of warmth along with episodic (later
blood pressure are higher in the upper extremities than the sustained) hypertension.
lower extremities. Femoral pulses are delayed and (u) B. Conn's Syndrome, also known as primary
weakened. Which of the following is the most likely hyperaldosteronism, is found in a patient with hypertension who
underlying diagnosis? has unprovoked hypokalemia.
A. Pheochromocytoma (u) C. Cushing's Syndrome is associated with hypertension but is
B. Conn's Syndrome associated with the typical appearance of sustained elevated cortisol
C. Cushing'sSyndrome levels such as purple striae, buffalo hump, and central obesity.
D. Coarctationoftheaorta (c) D. Coarctation of the aorta typically has narrowing of the aorta
proximal to the left subclavian artery with resultant high blood
pressure in the upper extremities and decreased run off to the
lower extremities following this narrowed segment.
164. Diagnostic Studies/Cardiology Explanations
Which of the following valvular heart abnormalities will (u) A. Aortic stenosis puts additional strain on the left ventricle and
most likely be seen on echocardiography as a complication contributes to a patient developing an acute myocardial infarction
of acute myocardial infarction? and does not occur as a result of one.
A. Aortic stenosis (u) B. Aortic regurgitation is not a consequence of acute myocardial
B. Aortic regurgitation infarction and most commonly occurs as a result of an incompetent
C. Mitral stenosis valve or dilation of the proximal aorta.
D. Mitral regurgitation (u) C. Mitral stenosis most commonly occurs as a complication of
rheumatic fever and not because of an acute myocardial infarction.
(c) D. In patients with acute myocardial infarction, echocardiogram
can show the severity of mitral regurgitation and the presence of
ventricular septal defect if one is present. Acute inferior wall
myocardial infarction is associated with acute mitral regurgitation
due to necrosis of the posterior papillary muscle which is supplied
by the right coronary artery.
165. History & Physical/Cardiology Explanations
Which of the following (c) A. Atrial septal defect can cause a left to right shunt with resultant increased volume in the
conditions is most closely right ventricle. The net result of this is that the P2 heart sound will be accentuated because of
associated with an increased the increased blood flow in the right ventricle and increased force of contraction in the right
intensity of the P2 heart sound? ventricle to remove this blood. Atrial septal defect will also cause a fixed split S2 heart sound.
A. Atrial septal defect (u) B. Aortic stenosis primarily affects the left ventricle and causes left ventricular hypertrophy.
B. Aortic stenosis As the left ventricle contracts harder against increased valvular resistance, a systolic ejection
C. Ventricular septal defect murmur occurs.
D. Mitral valve prolapse (u) C. Ventricular septal defect most commonly causes a holosystolic murmur as its most notable
ausculatory finding.
(u) D. Mitral valve prolapse most commonly is associated with a mid-systolic click with or without a
systolic heart murmur.
166. Clinical Therapeutics/Cardiology Explanations
Which of the following (u) A. Beta blockers do not cause vasodilation against this vasospasm which makes them less
medication classes is considered effective as treatment.
first-line therapy for patients (c) B. Calcium channel blockers are first line therapy for patients who have uncontrolled
with Raynaud's phenomenon? symptoms related to Raynaud's phenomenon. They are vasodilating agents which may play a
A. Beta blockers role in preventing the vasospasm that occurs with this disorder.
B. Calcium channel blockers (u) C. See B for explanation.
C. Central alpha agonists (u) D. Second line agents for Raynaud's phenomenon may include topical but not oral nitrates.
D. Oral nitrates
167. Scientific Concepts/Cardiology Explanations
Elevated levels of LP(a) (u) A. See C for explanation.
(Lipoprotein a) are considered to (u) B. See C for explanation.
be a risk factor for coronary (c) C. Lp(a) lipoproteins are secreted by the liver, constitute 10% or less of the total plasma
artery disease through which of lipoprotein mass, possess kringle domains homologous to plasminogen, and are associated with
the following proposed vascular disease risk. Having domains homologous to plasminogen, Lp(a) will compete with
mechanisms? actual plasminogen for its receptor sites. Plasminogen activates plasmin, which facilitates
A. Direct inhibition of HDL degradation of fibrin and matrix components. The main component of LP (a) is LDL, a known
B. Increasing the formation of risk factor for atherosclerosis.
VLDL cholesterol (u) D. See C for explanation.
C. Competes for binding to the
plasminogen receptor
D. Enhancement of naturally
circulating triglycerides
168. Clinical Therapeutics/Cardiology Explanations
Which of the following (u) A. Beta blockers need to be used with caution in a patient with severe left ventricular
medication classes is the dysfunction as they will worsen left ventricular contractility and may make this dysfunction
recommended treatment for worse. They are used, however, in the early stages of chronic heart failure.
patients who have an anterior (u) B. Calcium channel blockers have no proven mortality benefit in patients with myocardial
wall myocardial infarction with infarctions and left ventricular dysfunction.
poor left ventricular function? (u) C. Although potassium sparing diuretics are part of the later stage treatment of congestive
A. Beta blockers heart failure and tend to potentiate the other therapies, they are not first-line therapy in a
B. Calcium channel blockers patient with left ventricular dysfunction.
C. Potassium sparing diuretics (c) D. ACE inhibitors have been proven to be effective in the therapy of heart failure, especially
D. ACE inhibitors in the setting of left ventricular dysfunction. They are considered first-line therapy in patients
with symptomatic left ventricular systolic function.
169. Diagnosis/Cardiology Explanations
The 35 year-old patient presents after a syncopal episode while (u) A. Carotid sinus hypersensitivity typically presents with
throwing a football with his son. Examination reveals regular syncope that is related to turning of the head (such as backing a
heart rate and EKG is normal. There were no symptoms prior car out of a driveway) or from tight collars.
to the episode. Right radial pulse is decreased. Which of the (u) B. Vasovagal episodes may result in syncope but there would
following is the most likely explanation for the syncope? not be abnormal pulse findings.
A. Carotid sinus hypersensitivity (u) C. Cardiac dysrhythmia may occur in the setting of exercise
B. Vasovagal episode but this would not produce a decreased pulse on the affected
C. Cardiac dysrhythmia side.
D. Subclavian steal syndrome (c) D. Subclavian steal syndrome occurs if the subclavian artery is
occluded proximal to the origin of the vertebral artery which
results in reversal in the direction of blood flow in the ipsilateral
vertebral artery. Exercise of the ipsilateral arm may increase
demand on the vertebral flow which produces a "subclavian
steal".
170. Scientific Concepts/Cardiology Explanations
Which of the following is the most common complication that (c) A. Accumulation of transudate, exudate or blood in the
occurs in the setting of acute pericarditis? pericardial sac can occur due to pericardial inflammation.
A. Pericardial effusion (u) B. Patients with acute pericarditis may have problems with
B. Left ventricular failure filling which affects the right ventricle more than the left ventricle.
C. Superior vena cava syndrome (u) C. Patients with lung malignancy may develop superior vena
D. Subclavian steal syndrome cava syndrome as a result of tumor invasion into the superior
vena cava.
(u) D. Patients with subclavian steal syndrome typically present
with arm ischemia and syncope and is not related to pericarditis.
171. Diagnostic Studies/Cardiology Explanations
Which diagnostic study is considered to be the strategy of (h) A. Stress echocardiography should not be performed in the
choice for symptomatic patients with recurrent ischemia, setting of a patient who is acutely symptomatic.
hemodynamic instability or impaired left ventricular (h) B. Exercise treadmill testing should not be performed in the
dysfunction? setting of an unstable patient with ongoing cardiac symptoms.
A. Stress echocardiography (c) C. Coronary or cardiac catheterization is the gold standard
B. Exercise treadmill testing technique in the evaluation of patients with significant cardiac
C. Coronary angiography symptoms. Anatomical information along with degree of coronary
D. Cardiac magnetic resonance imaging artery blockages are provided and patients may be able to
undergo coronary revascularization during or after this
procedure.
(u) D. Cardiac magnetic resonance imaging has limited availability
and is not part of national guidelines for evaluation of the cardiac
patient.
172. History & Physical/Cardiology Explanations
Which of the following is a systemic manifestation of infective (u) A. Hemarthrosis is most commonly a consequence of a clotting
endocarditis? disorder such as hemophilia.
A. Hemarthrosis (c) B. Petechiae, splinter hemorrhages, Janeway lesions, and
B. Petechiae Osler's nodes are systemic manifestations of patients who have
C. Cafe au lait spots infective endocarditis.
D. Bronzing of the skin (u) C. Cafe au lait spots are seen in Neurofibromatosis (von
Recklinghausen's syndrome).
(u) D. Bronzing of the skin is most commonly associated with
hemochromatosis or Addison's disease.
173. Diagnostic Studies/Cardiology Explanations
A 76 year-old male presents after returning from a Safari in (u) A. Patients suffering from a non-STEMI myocardial infarction
Africa. Seven days ago he experienced chest pressure lasting will not develop Q waves and most likely will have a normal EKG
one hour that did not respond to three sublingual five days after an acute event.
nitroglycerin tablets. There was no ability to have lab work or (u) B. Myoglobulin is a nonspecific enzyme that is released into
an EKG. The pain has not returned. If the patient had a non- the circulation after any skeletal muscle damage, including a
STEMI myocardial infarction, which of the following studies myocardial infarction. It is the first enzyme that becomes positive
will still be positive? in the setting of acute myocardial infarction but its non-specific
A. Electrocardiogram measurement makes it less useful in the setting of acute
B. Myoglobulin myocardial infarction. It returns to baseline within 24 hours after
C. CK-MB index infarction.
D. Troponin I (u) C. CK-MB index has improved sensitivity for myocardial
muscle damage that occurs with acute myocardial infarction but
it returns to baseline within 2-3 days after injury.
(c) D. Troponin I levels will stay positive for at least one week
following myocardial infarction and is the preferred enzyme to
measure in this setting.
174. Clinical Intervention/Cardiology Explanations
A 52 year-old patient with episodes of syncope has an (c) A. This is consistent with ECG findings of a Mobitz type II AV
electrocardiogram which shows a consistently prolonged PR block. Since the patient is symptomatic this type of AV block
interval with a missing QRS every two beats. Which of the requires a permanent pacing to prevent total AV disassociation.
following is the most effective management? (u) B. Beta-blockers will slow conduction from the AV node and is
A. Permanent pacing not indicated with this type of AV block. (u) C. There is no
B. Beta-blocker indication for ACE Inhibitors in Mobitz Type II heart block.
C. ACEInhibitor (u) D. Defibrillation is not indicated in a person with AV block.
D. Defibrillation
175. Health Maintenance/Cardiology Explanations
According to the Joint National Commission VII Guidelines, (u) A. See B for explanation.
blood pressure targets are lower in patients with diabetes (c) B. Blood pressure targets for hypertensive patients at the
mellitus and what other condition? greatest risk for cardiovascular events, particularly those with
A. Liver disease diabetes and chronic kidney disease, are lower (less than
B. Renal disease 130/80) than for those individuals with lower cardiovascular risk
C. Thyroid disease (goal is less than 140/90).
D. Peripheral vascular disease (u) C. See B for explanation.
(u) D. See B for explanation.
176. Diagnosis/Cardiology Explanations
A patient presents to the office following a syncopal episode. (u) A. Carotid sinus hypersensitivity may present with syncope
The patient claims that the syncope occurs when he changes but is usually related to tight collars or when excessively turning
position such as rolling over in bed or when he bends over to the head.
tie his shoes. Which of the following is the most likely (u) B. Vasovagal episodes may occur with syncope as its
explanation for this presentation? manifestation but it is not caused by changes in position.
A. Carotid sinus hypersensitivity (u) C. Subclavian steal syndrome may present with syncope that
B. Vasovagal episode is related to exercise of the affected arm which results in a
C. Subclavian steal syndrome decreased pulse when the Adson maneuver is performed.
D. Atrial myxoma (c) D. Atrial myxoma most commonly presents with sudden
onset of symptoms that are typically positional in nature due to
the effect that gravity has on the tumor. Myxomas are the most
common type of primary cardiac tumor in all age groups and are
most commonly found in the atria.
177. Clinical Therapeutics/Cardiology Explanations
Which of the following is the optimal therapy for a 76 year-old (u) A. Aspirin's role to prevent thromboembolism in atrial
patient with no allergies who has chronic atrial fibrillation? fibrillation is limited to patients with no risk factors who are under
A. Aspirin age 65.
B. Clopidogrel (Plavix) (u) B. Clopidogrel is not the optimal therapy for patients with
C. Warfarin (Coumadin) atrial fibrillation.
D. Low molecular weight heparin (c) C. Patients older than age 75 who have chronic atrial
fibrillation should be anticoagulated with warfarin to maintain an
INR between 2.5 and 3.0 for optimum therapy unless a
contraindication to therapy exists.
(u) D. Due to the increased costs and need for parenteral
therapy, daily subcutaneous heparin is not first line therapy
unless warfarin therapy is contraindicated.
178. Clinical Intervention/Cardiology Explanations
Patients who undergo percutaneous angioplasty or who have (u) A. See B for explanation.
coronary artery revascularization often are treated with (c) B. Glycoprotein IIb/IIa inhibitors have their activity in the final
glycoprotein IIb/IIIa inhibitors. What is the major side effect stages of platelet bridging and are associated with bleeding when
associated with these agents? used in the management of acute myocardial infarction. Since
A. Hypotension they are effective at treating and preventing new clot formation,
B. Bleeding bleeding is the main concern and complication with the use of
C. Coronary vasospasm these agents.
D. Acute renal failure (u) C. See B for explanation.
(u) D. See B for explanation.
179. Health Maintenance/Cardiology Explanations
Which of the following is an absolute contraindication for the (u) A. See B for explanation.
performance of exercise stress testing for patients who wish to (c) B. Contraindications to stress testing include rest angina within
start an exercise program? the last 48 hours, unstable cardiac rhythm, hemodynamically
A. Second degree heart block type 1 unstable patient, severe aortic stenosis, acute myocarditis,
B. Severe aortic stenosis uncontrolled heart failure, and active infective endocarditis.
C. Atrial fibrillation with controlled ventricular response (u) C. See B for explanation.
D. Recent diagnosis of lung cancer (u) D. See B for explanation.
180. History & Physical/Cardiology Explanations
A 23 year-old male with recent upper respiratory symptoms (u) A. Acute pericarditis is usually not associated with elevated
presents complaining of chest pain. His pain is worse lying blood pressure. One would expect to see hypertensive pressures
down and better sitting up and leaning forward. in the setting of an aortic dissection.
Electrocardiogram shows widespread ST segment elevation. (u) B. Subungual hematomas are usually seen in endocarditis not
Which of the following is the most likely physical examination pericarditis.
finding in this patient? (u) C. A diastolic murmur in a patient with chest pain would likely
A. Elevated blood pressure be associated with acute aortic regurgitation in the setting of an
B. Subungual hematoma aortic dissection.
C. Diastolic murmur (c) D. This patient has symptoms consistent with acute pericarditis
D. Pericardial friction rub and would most likely have a pericardial friction rub on
examination.
181. Diagnostic Studies/Cardiology Explanations
A 53 year-old male with history of hypertension presents complaining of recent 4/10 (u) A. This patient has signs and
left-sided chest pain with exertion that is relieved with rest. He states the pain usually symptoms consistent with stable
lasts approximately 4 minutes and is relieved with rest. Heart examination reveals angina. Noninvasive diagnostic testing
regular rate and rhythm with no S3, S4, or murmur. Lungs are clear to auscultation is preferred in this patient.
bilaterally. Electrocardiogram reveals no acute changes. Which of the following is the (u) B. CT angiogram may be useful for
most appropriate initial step in the evaluation of this patient? the evaluation of chest pain, however
A. Cardiac catheterization its role in routine practice has not been
B. CT Angiogram of the chest established.
C. Echocardiogram (u) C. This patient has signs and
D. Nuclear stress test symptoms of stable angina. There are
no signs of valvular heart disease on
examination. While an echocardiogram
may be performed at some point, it is
not the best initial diagnostic step to
determine the etiology of the patient's
angina.
(c) D. Nuclear stress testing is the most
appropriate initial diagnostic study in
the evaluation of a patient with signs
and symptoms consistent with stable
angina.
182. Clinical Therapeutics/Cardiology Explanations
A 48 year-old male with diabetes mellitus presents for routine physical examination. Of (u) A. See D for explanation.
note his blood pressure each of his last two follow-up visits was 150/90 mmHg. Today (u) B. See D for explanation.
the patient's BP is 148/88 mmHg. The patient denies complaints of chest pain, change in (u) C. See D for explanation.
vision, or headache. Which of the following is the most appropriate management for (c) D. ACE inhibitors are the first line
this patient? treatment of choice in a patient with
A. Atenolol (Tenormin) hypertension and diabetes.
B. Nifedipine (Procardia)
C. Hydralazine (Apresoline)
D. Lisinopril (Zestril)
183. Health Maintenance/Cardiology Explanations
A 36 year-old female presents for a routine physical. She has no current complaints and (u) A. See B for explanation.
her only medication is oral contraceptives. The patient is preparing for a trip to (c) B. The risk of deep vein thrombosis
Australia and is worried about the long flight as her mom has a history of deep vein after air travel increases with flight
thrombosis after a long trip several years ago. Physical examination reveals BP 110/60 duration. Preventive measures for
mmHg, HR 66 bpm, regular. Heart is regular rate and rhythm without murmur, lungs patients include using support hose
are clear to auscultation bilaterally and extremities are without edema. Which of the and performing in-flight exercises and
following is the most appropriate recommendation for your patient? walking.
A. Discontinue oral contraceptives (u) C. See B for explanation.
B. Recommend walking frequently during the flight (u) D. See B for explanation.
C. Begin daily aspirin therapy
D. Increase fluid intake 2-3 days prior to the flight
184. Diagnostic Studies/Cardiology Explanations
A 3 month-old female presents with her mom for physical examination. The (u) A. This patient has signs and symptoms
patient's mom denies any complaints. On examination you note a well- consistent with a ventricular septal defect (VSD).
developed, well-nourished infant in no apparent distress. There is no CT angiogram and electrocardiogram are not
cyanosis noted. Heart examination reveals a normal S1 with a indicated in establishing the diagnosis of a VSD.
physiologically split S2. There is a grade III/VI high-pitched, harsh, (u) B. See A for explanation.
pansystolic murmur heard best at the 3rd and 4th left intercostal spaces (c) C. Echocardiogram is the initial diagnostic study
with radiation across the precordium. Which of the following is the initial of choice in the diagnosis of a VSD.
diagnostic study of choice in this patient? (u) D. Cardiac catheterization may be necessary to
A. CT angiogram accurately measure pulmonary pressures or if a
B. Electrocardiogram VSD can not be well localized on echocardiogram,
C. Echocardiogram but it is not the initial diagnostic study of choice.
D. Cardiac catheterization
185. Clinical Intervention/Cardiology Explanations
A 20 year-old male presents with complaint of brief episodes of rapid heart (u) A. Implantable cardio defibrillators are
beat with a sudden onset and offset that have increased in frequency. He indicated in the treatment of ventricular
admits to associated shortness of breath and lightheadedness. He denies arrhythmias, not Wolf-Parkinson- White (WPW)
syncope. Electrocardiogram reveals a delta wave prominent in lead II. Which syndrome.
of the following is the most appropriate long-term management in this (c) B. Radiofrequency ablation is the procedure of
patient? choice for long-term management in patients with
A. Implantable cardio defibrillator accessory pathways (WPW) and recurrent
B. Radiofrequency ablation symptoms.
C. Verapamil (Calan) (u) C. Calcium channel blockers and beta-blockers
D. Metoprolol (Lopressor) are not the best options for the long-term
management of WPW. They may decrease the
refractoriness of the accessory pathway or
increase the refractoriness of the AV node in
patients with atrial fibrillation or atrial flutter who
have an antegrade conducting bypass tract. This
may lead to faster ventricular rates.
(u) D. See C for explanation.
186. Diagnosis/Cardiology Explanations
A 60 year-old female recently discharged after an 8 day hospital stay for (u) A. Acute thromboembolism is usually
pneumonia presents complaining of pain and redness in her right arm. The associated with edema of the extremity and warm
patient thinks this was the area where her IV was placed. The patient denies temperature. Thromboembolism is unusual after
fever or chills. Examination of the area reveals localized induration, thrombophlebitis.
erythema and tenderness. There is no edema or streaking noted. Which of (c) B. This patient's signs and symptoms are
the following is the most likely diagnosis? consistent with thrombophlebitis. Short-term
A. Acute thromboembolism venous catheterization of a superficial arm vein is
B. Thrombophlebitis commonly the cause and thrombophlebitis
C. Cellulitis characterized by dull pain, induration, redness
D. Lymphangitis and tenderness along the course of the vein.
(u) C. Cellulitis is usually associated with fever,
increased warmth over the affected area and
associated edema. (u) D. Lymphangitis is
associated with fever, malaise, chills, and streaking.
187. History & Physical/Cardiology Explanations
A patient with a history of chronic venous insufficiency presents for routine (u) A. Cold lower extremities are more commonly
follow-up. Which of the following findings is most likely on physical seen in peripheral arterial, not venous, disease.
examination? (u) B. Diminished pulses are seen in peripheral
A. Cold lower extremities arterial disease.
B. Diminished pulses (c) C. Patients with chronic venous insufficiency will
C. Lower extremity edema commonly have lower extremity edema.
D. Palpable cord (u) D. A palpable cord is more common in
superficial thrombophlebitis.
188. Diagnostic Studies/Cardiology Explanations
A 29 year-old female with history of IV drug abuse presents with ongoing fevers for three weeks. She (u) A. See D for
complains of fatigue, worsening dyspnea on exertion and arthralgias. Physical examination reveals a explanation.
BP of 130/60 mmHg, HR 90 bpm, regular, RR 18, unlabored. Petechiae are noted beneath her (u) B. See D for
fingernails. Fundoscopic examination reveals exudative lesions in the retina. Heart examination explanation.
shows regular rate and rhythm, there is a grade II-III/VI systolic murmur noted, with no S3 or S4. (u) C. See D for
Lungs are clear to auscultation bilaterally, and the extremities are without edema. Which of the explanation.
following is the diagnostic study of choice in this patient? (c) D. This patient's
A. Electrocardiogram signs and symptoms
B. CT angiogram of the chest are consistent with
C. Cardiac catheterization infective endocarditis.
D. Transesophageal echocardiogram The diagnostic study of
choice would be a
transesophageal
echocardiogram.
189. Clinical Therapeutics/Cardiology Explanations
A 49 year-old female presents complaining of several episodes of chest pain recently. Initial ECG in the (c) A. This patient is
emergency department shows no acute changes. Two hours later, while the patient was having pain, most likely having
repeat electrocardiogram revealed ST segment elevation in leads II, III, and AVF. Cardiac coronary artery spasm.
catheterization shows no significant obstruction of the coronary arteries. Which of the following is This can be treated
the treatment of choice in this patient? prophylactically with
A. Nifedipine (Procardia) calcium channel
B. Metoprolol (Lopressor) blockers such as
C. Lisinopril (Zestril) nifedipine.
D. Carvedilol (Coreg) (h) B. Beta-blockers
may exacerbate the
symptoms of coronary
vasospasm.
(u) C. ACE inhibitors
are not effective in the
treatment or
prevention of coronary
vasospasm.
(u) D. Carvedilol is not
effective in the
treatment or
prevention of coronary
vasospasm.
190. History & Physical/Cardiology Explanations
A 75 year-old female with a history of long-standing hypertension presents (c) A. This patient has history findings consistent
with shortness of breath. On examination you note a diastolic murmur at with aortic insufficiency which is characterized by a
the left upper sternal border. Which of the following maneuvers would diastolic murmur that is accentuated when the
accentuate this murmur? patient sits up and leans forward.
A. Sitting up and leaning forward (u) B. The left lateral decubitus position
B. Lying on left side accentuates the murmur of mitral stenosis.
C. Performing Valsalva maneuver (u) C. Valsalva and standing maneuvers help to
D. Standing upright differentiate the murmurs associated with aortic
stenosis and hypertrophic cardiomyopathy.
(u) D. See C for explanation.
191. Diagnosis/Cardiology Explanations
A 50 year-old male with history of alcohol abuse presents with complaint of (u) A. Hypertrophic cardiomyopathy is
worsening dyspnea. Physical examination reveals bibasilar rales, elevated characterized by a hyperdynamic left ventricle
jugular venous pressure, an S3 and lower extremity edema. Chest x-ray with asymmetric left ventricular hypertrophy.
reveals pulmonary congestion and cardiomegaly. Electrocardiogram shows (c) B. Dilated cardiomyopathy is often caused by
frequent ventricular ectopy. Echocardiogram shows left ventricular chronic alcohol use. It is characterized by signs and
dilatation and an ejection fraction of 30%. Which of the following is the symptoms of left-sided heart failure, a dilated left
most likely diagnosis in this patient? ventricle and decreased ejection fraction.
A. Hypertrophic cardiomyopathy (u) C. Restrictive cardiomyopathy is characterized
B. Dilated cardiomyopathy more commonly by right-sided heart failure than
C. Restrictive cardiomyopathy by left-sided heart failure. There is rapid early
D. Tako-Tsubo cardiomyopathy filling with diastolic dysfunction. Patients with
restrictive cardiomyopathy will have a small
thickened left ventricle and a normal or near
normal ejection fraction on echocardiogram.
(u) D. Tako-Tsubo cardiomyopathy (broken heart
syndrome) is characterized by signs and
symptoms of acute coronary syndrome, ST
segment elevation on ECG and left ventricular
apical dyskinesia.
192. Clinical Therapeutics/Cardiology Explanations
A 76 year-old active female with history of hypertension and (c) A. Anticoagulation is necessary in all patients
hypothyroidism presents with complaints of palpitations and dyspnea on with atrial fibrillation to prevent thromboembolic
exertion. On examination vital signs are BP 120/80 mmHg, HR 76 bpm, events unless there is contraindication.
irregular, RR 16. Heart examination reveals an irregularly, irregular rhythm (u) B. This patient currently has a controlled
without murmur. Lungs are clear to auscultation and extremities are ventricular rates and does not require chronic
without edema. Which of the following is the most important medication calcium channel blockers or digoxin at this time.
to initiate for chronic therapy in this patient? (u) C. Antiarrhythmic therapy may be indicated in
A. Warfarin (Coumadin) some patients with atrial fibrillation, but
B. Verapamil (Calan) anticoagulation is indicated in all patients unless
C. Amiodarone (Cordarone) there is contraindication.
D. Digoxin (Lanoxin) (u) D. See B for explanation.
193. Diagnosis/Cardiology Explanations
A 58-year old male presents for a six week follow-up after an acute (c) A. Left ventricular (LV) aneurysm develops in about 10-
anterior wall myocardial infarction. He denies chest pain and 20 percent of patients following acute myocardial
shortness of breath. Electrocardiogram shows persistent ST segment infarctions, especially anterior wall myocardial infarctions.
elevation in the anterior leads. Echocardiogram reveals a sharply LV aneurysm is identified by ST segment elevation that is
delineated area of scar that bulges paradoxically during systole. present beyond 4-8 weeks after the acute infarct and a
Which of the following is the most likely diagnosis in this patient? scar that bulges paradoxically during systole on
A. Left ventricular aneurysm echocardiogram.
B. Postinfarction ischemia (u) B. Postinfarction ischemia is recurrent ischemia that is
C. Ischemic cardiomyopathy more common in patients with non-ST segment elevation
D. Constrictive pericarditis myocardial infarctions and is characterized by
postinfarction angina. This patient denies any chest pain.
(u) C. Ischemic cardiomyopathy would be characterized by
decreased ejection fraction on echocardiogram and wall
motion abnormalities. Ischemic cardiomyopathy is not
associated with ST segment elevation or bulge of scar on
echocardiogram.
(u) D. Constrictive pericarditis is characterized by signs
and symptoms of right-sided heart failure with increased
jugular venous pressures and a septal bounce on
echocardiogram.
194. History & Physical/Cardiology Explanations
A 75 year-old female with history of coronary artery disease and (u) A. This patient has signs and symptoms consistent with
dyslipidemia presents for routine follow-up. Physical examination arterial insufficiency. Lower extremity edema is seen in
reveals loss of hair on the lower extremities bilaterally with thinning patients with venous insufficiency.
of the skin. Femoral pulses are +2/4 bilaterally, pedal pulses are (c) B. This patient has signs and symptoms consistent with
diminished bilaterally. Ankle brachial index is reduced. Which of the arterial insufficiency and would most likely complain of
following signs or symptoms is this patient most likely to have? intermittent claudication.
A. Lower extremity edema (u) C. Numbness of the lower extremities would be seen
B. Calf pain with walking with acute arterial occlusion.
C. Numbness of the lower extremities (u) D. Itching of the lower extremities may be seen in
D. Itching of the lower extremities chronic venous insufficiency because of secondary skin
changes, but is not common in arterial insufficiency.
195. Diagnosis/Cardiology Explanations
A 55 year-old male with history of hypertension and diabetes (u) A. Resistant hypertension is the failure to reach blood
mellitus presents to the emergency department. The patient's wife pressure control in patients who are compliant with a 3
states that the patient developed progressive irritability and drug regimen including a diuretic.
confusion today after complaining of a headache. Physical (u) B. Hypertensive urgency is a systolic BP > 220 or a
examination reveals a BP of 230/130 mmHg and papilledema. Which diastolic BP > 125 in a patient who is asymptomatic or who
of the following is the most accurate diagnosis in this patient? has disk edema, progressive target organ complications.
A. Resistant hypertension Hypertensive urgency must be treated within a few hours
B. Hypertensive urgency of presentation.
C. Hypertensive emergency (u) C. Hypertensive emergency is similar to hypertensive
D. Malignant hypertension urgency, however the BP is significantly elevated and must
be lowered within an hour.
(c) D. Malignant hypertension is significantly elevated BP
with progressive retinopathy, including papilledema,
encephalopathy, and headache.
196. Scientific Concepts/Cardiology Explanations
A patient undergoes biopsy for suspected myocarditis. Which of the (u) A. See C for explanation.
following is the most likely etiologic agent? (u) B. See C for explanation.
A. West Nile virus (c) C. Coxsackie B virus, Hepatitis C, adenovirus, and HIV
B. Rhinovirus are the predominant agents in clinically significant acute
C. Coxsackie B virus viral myocarditis in the US.
D. Cytomegalovirus (u) D. See C for explanation.
197. Health Maintenance/Cardiology Explanations
A 65 year-old male with coronary artery disease, hypertension, and diabetes mellitus is (u) A. Elevation of the head of the
admitted with dyspnea and lower extremity edema. The chest x-ray reveals small bilateral bed may help the patient if they
pleural effusions. Echocardiogram shows an ejection fraction of 30% with no valvular heart have symptoms of dyspnea but it
disease. The patient is treated in the hospital with furosemide (Lasix) and lisinopril (Zestril). will not help prevent readmission
What education should be given to this patient upon discharge to help prevent to the hospital with a CHF
readmission? exacerbation.
A. Elevate the head of bed at home (u) B. In stable patients increasing
B. Avoid physical activity physical activity or regular
C. Monitor daily weights exercise can diminish symptoms.
D. Restrict fluid intake (c) C. Strategies to prevent
rehospitalization can include
monitoring daily weights, case
management and patient
education regarding self-
adjustment of diuretics.
(u) D. Fluid restriction is not
helpful in the readmission for
CHF.
198. History & Physical/Cardiology Explanations
A 60 year-old male nonsmoker with history of coronary artery disease presents with (u) A. A pericardial friction rub is a
complaint of worsening dyspnea on exertion for three weeks. He admits to orthopnea and sign of pericarditis, not heart
lower extremity edema, but denies chest pain, palpitations, and syncope. The patient's last failure.
echocardiogram revealed an ejection fraction of 30%. Which of the following would you (c) B. An S3 on physical
most likely find on physical examination? examination is consistent with
A. Pericardial friction rub heart failure.
B. Third heart sound (u) C. An accentuated first heart
C. Accentuated first heart sound sound is noted in tachycardia,
D. Mid-systolic click short PR interval rhythms,
increased cardiac output states
and mitral stenosis.
(u) D. A mid-systolic click is noted
in patients with mitral valve
prolapse.
199. Clinical Therapeutics/Cardiology Explanations
A 78 year-old male with history of coronary artery disease s/p coronary artery bypass (c) A. Sublingual nitroglycerine is
grafting, hypertension, and dyslipidemia presents for routine physical examination. He feels the drug of choice for the acute
well except for occasional brief episodes of substernal chest pain with exertion that are management of chronic stable
relieved with rest. He denies associated dyspnea, nausea or diaphoresis. Physical angina.
examination reveals a BP of 110/70 mmHg, HR 56 bpm, regular, RR 14, unlabored. Lungs are (u) B. Beta-blockers are
clear to auscultation, heart is bradycardic, but regular with no S3, S4 or murmur. preventative and not the first
Electrocardiogram done in the office shows no acute ST-T wave changes. Which therapy is choice for the acute management
indicated for the acute management of this patient's symptoms? of chronic stable angina. Beta-
A. Sublingual nitroglycerine blockers may worsen this
B. Metoprolol (Lopressor) patient's bradycardia.
C. Verapamil (Calan) (u) C. Calcium channel blockers
D. Lisinopril (Zestril) are the third-line antiischemic
agent and may also reduce the
patient's heart rate.
(u) D. ACE inhibitors will not
provide acute relief of anginal
symptoms.
200. Scientific Concepts/Cardiology Explanations
A 36 year-old male presents for follow-up of his hypertension. The patient is currently on (u) A. Pheochromocytoma is an
three anti-hypertensive medications without improvement of his blood pressure. On uncommon cause of
examination his BP is 170/86mmHg and his HR is 60bpm and regular. His heart examination hypertension characterized by
reveals a regular rate and rhythm without S3, S4 or murmur and his lungs are clear to paroxysms of headache, sweating
auscultation bilaterally. Abdominal examination reveals a bruit over his left upper and palpitations. There are no
abdomen. Which of the following is the most likely underlying etiology for this patient's bruits associated with
hypertension? pheochromocytoma.
A. Pheochromocytoma (c) B. Renal artery stenosis is
B. Renal artery stenosis characterized by hypertension
C. Cushing syndrome that is resistant to three or more
D. Coarctation of the aorta medications and renal artery
bruits on examination.
(u) C. Cushing syndrome is
characterized by "moon" facies, a
buffalo hump, a protuberant
abdomen, weakness and
headache. There are no renal
artery bruits associated with
Cushing syndrome.
(u) D. Coarctation of the aorta is
associated with hypertension in
the upper extremities and
normal or low blood pressure in
the lower extremities. There are
often weak femoral pulses and a
late systolic ejection murmur or
associated aortic insufficiency
murmur.
201. Clinical Therapeutics/Cardiology Explanations
A 16 year-old male presents with complaint of syncope after basketball practice today. (c) A. Beta-blockers are the
Physical examination reveals a systolic murmur along the left sternal border that increases initial drug of choice in a
with Valsalva maneuver. An electrocardiogram reveals left ventricular hypertrophy. symptomatic patient with
Echocardiogram shows asymmetric left ventricular hypertrophy with a hypercontractile left hypertrophic cardiomyopathy.
ventricle. Which of the following is the initial medication of choice in this patient? (u) B. See A for explanation.
A. Metoprolol (Lopressor) (u) C. See A for explanation.
B. Losartan (Cozaar) (u) D. See A for explanation.
C. Lisinopril (Zestril)
D. Hydrochlorothiazide (Diuril)
202. Clinical Intervention/Cardiology Explanations
An 80 year-old female presents with syncope and recent fatigue and lightheadedness over the (u) A. Cardio defibrillators treat
past month. She denies chest pain or dyspnea. Physical examination reveals BP 130/70 mmHg, ventricular tachycardia and
HR 40 bpm, regular, and RR 16. Electrocardiogram reveals two p waves before each QRS are not indicated in the
complex. Which of the following is the treatment of choice for this patient? management of second degree
A. Cardio defibrillator insertion AV block.
B. Atropine as needed (u) B. Atropine can be used in
C. Permanent dual chamber pacemaker insertion the acute management of
D. Ritalin therapy daily second degree AV block Mobitz
type II, but it should not be
used as long-term therapy.
(c) C. This patient has findings
consistent with symptomatic
second degree AV block Mobitz
type II for which permanent
pacing is the treatment of
choice.
(u) D. Ritalin therapy is not
indicated in the management
of second degree heart block.
203. History & Physical/Cardiology Explanations
Which of the following would be expected on physical examination of a newborn diagnosed (c) A. Tetralogy of Fallot is
with Tetrology of Fallot? commonly associated with a
A. Palpable right ventricular lift palpable right ventricular lift.
B. Pulsediscrepancybetweenarmsandlegs (u) B. Coarctation of the aorta is
C. Mid-diastolic murmur with opening snap associated with a pulse
D. Polymorphous exanthema discrepancy between the
upper and lower extremities.
(u) C. A mid-diastolic murmur
with an opening snap is heard
in a patient with mitral stenosis,
not Tetralogy of Fallot.
(u) D. Polymorphous
exanthema is seen in patients
with Kawasaki disease.
204. Clinical Therapeutics/Cardiology Explanations
A 67 year-old male with history of mitral valve stenosis undergoes a mechanical valve (u) A. See D for explanation.
replacement. Which of the following is the appropriate duration of anticoagulation therapy if (u) B. See D for explanation.
the patient has no other risk factors for thromboembolic events or significant bleeding risks? (u) C. See D for explanation.
A. One month (c) D. Patients with mechanical
B. Three months valves require lifelong
C. Six months anticoagulation to prevent
D. Lifelong thrombosis.
205. Diagnosis/Cardiology Explanations
A 60 year-old female with history of radiation therapy for the treatment of (u) A. Pulmonary hypertension is usually associated
cancer presents with progressive dyspnea and fatigue. On examination the with chest pain, dyspnea, fatigue and syncope.
patient has lower extremity edema, significant ascites, and an elevated Examination would reveal a narrow splitting of S2
jugular venous pressure that does not fall with inspiration. Heart with a loud pulmonic component. Echocardiogram
examination reveals a pericardial knock. Echocardiogram shows rapid would show increased pulmonary artery pressures,
early filling and reduced mitral inflow velocities with inspiration. Which of right ventricular enlargement and possibly
the following is the most likely diagnosis in this patient? paradoxical motion of the intraventricular septum.
A. Pulmonaryhypertension (u) B. Atrial myxoma is characterized by fever,
B. Atrialmyxoma weight loss, malaise, embolization, and a diastolic
C. Constrictive pericarditis sound or murmur (tumor plop) on examination.
D. Tako-Tsubo cardiomyopathy Atrial myxoma would be seen on echocardiogram.
(c) C. Constrictive pericarditis is associated with TB,
radiation therapy, cardiac surgery, or following viral
pericarditis. There is evidence of right-sided heart
failure, a positive Kussmaul sign, and a septal
bounce and reduced mitral inflow velocities with
inspiration on echocardiogram.
(u) D. Tako-Tsubo cardiomyopathy (broken heart
syndrome) is commonly seen in postmenopausal
women who experience signs and symptoms of
acute coronary syndrome. Echocardiogram shows
left ventricular apical dyskinesia.
206. Scientific Concepts/Cardiology Explanations
A 59 year-old male with history of hypertension and dyslipidemia presents (u) A. See B for explanation.
with complaint of substernal chest pain for two hours. The pain woke him (c) B. Inferior wall myocardial infarction is
from sleep, does not radiate, and is associated with nausea and characterized by ST segment elevation in leads II, III
diaphoresis. Electrocardiogram reveals ST segment elevation in leads II, III, and AVF.
and AVF. Which of the following walls of the ventricle is most likely at risk? (u) C. See B for explanation.
A. Anterior (u) D. See B for explanation.
B. Inferior
C. Lateral
D. Posterior
207. Diagnosis/Cardiology Explanations
A 24 year-old female presents complaining of palpitations described as (c) A. Patients with mitral valve prolapse will often
occasional "skipped" beats. The patient denies chest pain, present with complaint of palpitations. Auscultation
lightheadedness, syncope, or dyspnea. On examination you note a would reveal a mid-systolic click with or without a
midsystolic click without murmur. Which of the following is the most late systolic murmur.
likely diagnosis in this patient? (u) B. Aortic stenosis presents with a systolic
A. Mitral valve prolapse murmur with no click. Patients may complain of
B. Aortic stenosis chest pain, dyspnea or syncope.
C. Atrial septal defect (u) C. Atrial septal defect is not associated with a
D. Pulmonic stenosis midsystolic click.
(u) D. Pulmonic stenosis is not associated with a
midsystolic click.
208. Health Maintenance/Cardiology Explanations
A 10 year-old female experiences fever and polyarthralgia. On examination you (u) A. Doxycycline and Bactrim are not
note a new early diastolic murmur. Laboratory results are positive for indicated for the prophylaxis of recurrent
antistreptolysin O. The patient has no known drug allergies. Which of the rheumatic fever.
following is the recommended prophylaxis for this condition? (u) B. Erythromycin is considered second line
A. Doxycycline for prophylaxis of recurrent rheumatic fever
B. Erythromycin in a patient with a penicillin allergy.
C. BenzathinepenicillinG (c) C. Recurrences of rheumatic fever are
D. Trimethoprim/sulfamethoxazole most common in patients who have had
carditis during their initial episode and in
children. The preferred method of
prophylaxis is Benzathine penicillin G every
four weeks.
(u) D. See A for explanation.
209. Clinical Intervention/Cardiology Explanations
A 26 year-old patient is brought to the emergency department after a head on (u) A. Serial echocardiograms would be
collision. The patient complains of chest pain, dyspnea and cough. Examination indicated if a patient had a small pericardial
reveals the patient to be tachypneic and tachycardic with a narrow pulse effusion and no intervention was
pressure. Jugular venous distension is noted. Electrocardiogram reveals immediately needed. This patient has signs
nonspecific t wave changes and electrical alternans. Which of the following is the and symptoms of cardiac tamponade and
most appropriate management plan for this patient? needs immediate intervention.
A. Serial echocardiograms (c) B. Urgent pericardiocentesis is the initial
B. Pericardiocentesis treatment of choice in a patient with cardiac
C. Cardiac catheterization tamponade.
D. Pericardiectomy (u) C. There is no indication for cardiac
catheterization in the management of cardiac
tamponade.
(u) D. A partial pericardiectomy may be
needed in patients with recurrent pericardial
effusions that occur secondary to neoplastic
disease and uremia, but there is no
indication for partial pericardiectomy in the
acute management of cardiac tamponade.
210. Health Maintenance/Cardiology Explanations
A 40 year-old G3P3003 female presents complaining of dull aching discomfort of (c) A. Graduated compression stockings can
her lower extremities, which is worse in the evening. The patient currently works be used in patients with early varicosities to
as a waitress. Examination reveals dilated, tortuous veins beneath the skin in the prevent progression of the disease and
thigh and leg bilaterally. Which of the following is the best initial approach to when used with leg elevation complications
prevent progression of disease and complications in this patient? from varicose veins can be avoided.
A. Compression stockings (u) B. There is no indication for warfarin or
B. Warfarin (Coumadin) therapy clopidogrel therapy in patins with varicose
C. Sclerotherapy veins.
D. Clopidogrel (Plavix) (u) C. Scierotherapy is not the best initial
choice to prevent disease progression and
complications
(u) D. See B for explanation.
211. Diagnostic Studies/Cardiology Explanations
A 60 year-old male with history of hypertension presents for routine physical (u) A. MRI, arteriography, and
examination. He has no current complaints. Vital signs are BP of 136/70 mmHg, HR 60 abdominal flat plate are not indicated
bpm, regular, RR 14, unlabored. Heart shows regular rate and rhythm with no S3, S4 or in the initial diagnostic evaluation of a
murmur, Lungs are clear to auscultation bilaterally, and the abdomen is soft, patient with a suspected abdominal
nontender. There is a 5cm palpable pulsatile abdominal mass noted. Which of the aortic aneurysm.
following is the best initial diagnostic study in this patient? (u) B. See A for explanation.
A. Magnetic resonance imaging (MRI) (c) C. Abdominal ultrasound is the
B. Arteriography diagnostic study of choice for the initial
C. Ultrasound (US) diagnosis of an abdominal aortic
D. Plain film aneurysm.
(u) D. See A for explanation.
212. Diagnosis/Cardiology Explanations
A 70 year-old female with history of hypertension, diabetes, and hypothyroidism (c) A. Acute arterial occlusion presents
presents with complaint of sudden onset of left lower extremity pain. Examination with sudden onset of extremity pain,
reveals a cool left lower extremity with a mottled appearance. Dorsalis pedis and with absent or diminished pulses. The
posterior tibialis pulses are absent. Which of the following is the most likely diagnosis? extremity will be cool to the touch and
A. Acute arterial occlusion have a mottled appearance.
B. Thromboangiitis obliterans (u) B. Thromboangitis obliterans occurs
C. Deep vein thrombosis in younger patients and primarily
D. Peripheral neuropathy effects the distal extremities, especially
the toes. It is typically secondary to
smoking.
(u) C. Deep vein thrombosis presents
with lower extremity pain and edema.
Pulses would be intact.
(u) D. Patients with peripheral
neuropathy would have diminished
sensation. They would not have a
mottled appearance and pulses would
be intact.
213. A 45 year-old male presents to the Emergency Department complaining of sudden Explanations
onset of tearing chest pain radiating to his back. On examination the patient is (c) A. This patient has signs and
hypertensive and his peripheral pulses are diminished. Electrocardiogram shows no symptoms of acute aortic dissection for
acute ST-T wave changes. Which of the following is the diagnostic study of choice in which CT scan is the diagnostic study of
this patient? choice.
A. Computed tomography (CT) scan (u) B CT scan is better than
B. Transthoracic echocardiogram transthoracic echocardiogram for the
C. Magnetic resonance imaging (MRI) diagnosis of acute aortic dissection.
D. Cardiac catheterization Transesophageal echocardiogram (TEE)
is a good diagnostic modality, however
it is not always available in the acute
setting.
(u) C.MRI is good in the diagnosis of a
chronic aortic dissection, but the longer
imaging time and the difficulty in
monitoring the patient during the test
makes it not the first choice in the
setting of an acute dissection.
(u) D. Cardiac catheterization is not
indicated in the diagnosis of an acute
aortic dissection.
214. Scientific Concepts/Cardiology Explanations
A 60 year-old male complains of progressive fatigue and dyspnea. On examination his lungs are (u) A. Chronic alcohol use is
clear to auscultation bilaterally, heart exam reveals regular rate and rhythm without S3, S4 or commonly associated with a
murmur, and extremities show 1+ edema bilaterally. Chest x-ray reveals cardiomegaly. dilated left ventricle with left
electrocardiogram shows low voltage, and echocardiogram shows an ejection fraction of 55% ventricular dysfunction.
with a small, thickened left ventricle that has rapid early filling with diastolic dysfunction. (u) B. Myocarditis is associated
Which of the following is the most likely underlying etiology of this patient's cardiomyopathy? with a dilated, not small, left
A. Alcoholism ventricle.
B. Myocarditis (c) C. Amyloidosis is the most
C. Amyloidosis common cause of restrictive
D. Chronic hypertension cardiomyopathy and is
associated with a small
thickened left ventricle that
has rapid early filling with
diastolic dysfunction.
(u) D. Chronic hypertension is
associated with a
hypertrophic,
hypercontractile left ventricle.
215. Clinical Intervention/Cardiology Explanations
A 70 year-old male with history of ischemic cardiomyopathy presents with a syncopal episode. (u) A. See C for explanation.
He denies complaints of chest pain, palpitations, or dyspnea. ECG shows no acute ST-T wave (u) B. See C for explanation.
changes. Echocardiogram reveals an ejection fraction of 25% with no valvular abnormalities. (c) C. This patient has ischemic
Which of the following is the most appropriate management for this patient? cardiomyopathy and syncope,
A. Dual chamber permanent pacemaker which is most likely due to
B. Diltiazem (Cardizem) ventricular tachycardia.
C. Implantable cardio defibrillator Instertion of a cardio
D. Midodrine (ProAmatine) defibrillator is the
management of choice in this
patient.
(u) D. See C for explanation.

Vous aimerez peut-être aussi